You are on page 1of 202

Colen Publishing

www colenpublishing com


www.colenpublishing.com
©™
Colen Publishing
Colen Publishing, L.L.C.
PO Box 36536
Grosse Pointe Woods, MI 48236
Author and Editor: Chaim B. Colen, M.D., Ph.D.
Editorial Assistant: Roxanne E. Colen, PA
PA-C
C

COPYRIGHT © 2008 by Colen Publishing, L.L.C. This book, including all parts thereof, is legally protected by copyright. Any use, exploitation, or commercialization outside the narrow
limits set by copyright legislation without the author’s consent if illegal and liable to prosecution. This applies in particular to photostat reproduction, copying, mimeographing or
duplication of any kind, translating, preparation of microfilms, and electronic data processing and storage.
Some of the product names, patents, and registered designs referred to in this book are in fact registered trademarks or proprietary names even though specific reference to this fact
is not always made in the text. Therefore, the appearance of a name without designation as proprietary is not to be construed as a representation by the publisher that it is in the
public domain. Permissions may be sought directly from Colen Publishing, L.L.C. by writing to the above address.
Printed in China PakPal:
ISBN 9781935345091 1935345095 Colen FlashReview: NPath 2009
9781935345107 1935345109 Colen FlashReview: Nlgy 2009
Colen Flash-Review: Neurology
9781935345114 1935345117 Colen FlashReview: NAn 2009
Volume 1: 0-9788502-4-6 9781935345121 1935345125 Colen FlashReview: NCC 2009
Volume 2: 0-9788502-5-4 9781935345138 1935345133 Colen FlashReview: NRad 2009
2 Volume Set: 0-9788502-9-7 9781935345145 1935345141 Colen FlashReview: NBio 2009
9781935345152 193534515X Colen FlashReview: NBeh 2009
Note: Knowledge in medicine is constantly changing. The author has consulted sources believed to be reliable in the effort to provide information that is complete and in accord with
the standards at the time of publication. However, in view of the possibility of human error by the author in preparation of this work, warrants that the information contained herein is in
every respect accurate and complete, and that the author is not responsible for any errors or omissions or for the results obtained from use of such information. The reader is advised
to confirm the information contained herein with other sources
sources. This is especially important in connection with new or infrequently used drugs.
drugs In such instances,
instances the product
information sheet included in the package with each drug should be reviewed.
©™
Glossary

COPYRIGHT-------------------------------------------------- 1
PREFACE ------------------------------------------------------ 1
HOW TO USE THIS CARD REVIEW-------------------- 1
CONTRIBUTORS-------------------------------------------- 4
GLOSSARY--------------------------------------------------- 1
NEUROSURGERY------------------------------------------ 110
NEUROLOGY ------------------------------------------------ 86
NEUROPATHOLOGY-------------------------------------- 238
NEUROANATOMY----------------------------------------- 57
NEUROCRITICAL CARE---------------------------------- 80
NEURORADIOLOGY--------------------------------------- 73
NEUROBIOLOGY
NEUROBIOLOGY------------------------------------------- 64
©™
BONUS BIOSTATISTICS---------------------------------- 6
Preface
• The idea to undertake such a large Flashcard review spawned from watching my wife Roxanne
study for her Physician Assistant Boards. Diligently every day she would create a set of 7-10
flashcards from her studyy material that she would take with her to work. Later on,, when I was
studying for my written Neurosurgery Board examination, I gleaned information from various
texts and other study guides and wrote down the most relevant material on cards for quick
review while at work. It was amazing how much time during the day would be available to review
these cards. If there was a delay in a OR case, a long lunch-line, a traffic jam (especially the i94
on a Friday afternoon) or waiting for my wife at her OB/GYN appointment -these little cards were
specially handy. Always ambitious in life, the thought of giving this study tool to the busy
neurosurgery resident was captivating. My expectation is to enable the resident with a quick yet
informative review of basic neuroscience principles. With positive encouragement from my fellow
residents on the 1st edition, I cautiously proceed here with updating information, adding new
images, improved illustrations and clarification of neuroscience concepts. May this endeavor
serve to better our wonderful science inherited through the legacy of Harvey Cushing,
©™
Neurosurgery.
Chaim September 9, 2008
The Colen Flash-Review
Flash Review
Author and Editor
Chaim B. Colen, M.D., Ph.D.
Department of Neurological Surgery
Wayne State University School of Medicine
Detroit, Michigan

Assistant Editor
Roxanne E. Colen, M.S., PA-C
Colen Publishing, LLC
Grosse Pointe, Michigan

©™
Acknowledgements
I would like to give thanks to a great many wonderful persons whose efforts, although not inscribed in
these cards, were instrumental in making this monumental task possible. One exceptional individual to whom I
owe special thanks is my mother in-in-law, Colleen Johns, who babysat my daughter Emily and son Joshua for
hours on end,
end while my wife and I toiled through hundreds of pages of various textbooks and journal articles
articles,
formatted questions, and drew computer illustrations. To my daughter Emily Rivka, who incessantly tugged at
my pants trying to get my attention to the squirrel in our backyard ;and that big bright smile from my son Joshua
that continually sent me optimism. To Mahmoud and Abhi who spent hours at my home assisting with typing,
researching and editing; Naomi whose positive attitude in life is exceptionally brightening and uplifted the
group’s 2 am brainstorming sessions when I still had to wake up early to work the next day, all the pathologists,
especially Doha, who assisted in taking photographs, Dr. William Kupsky, for allowing us access to his collection
of unique neuropathology, and to all the medical students especially Kristyn, whose hard work is admirable.
There are those whose names are not here but did assist in some way, thank you. I am forever indebted to my
training program, the Wayne State University neurosurgery program, my Chairman Dr. Murali Guthikonda, and
Associate Chairman Dr. Setti S. Rengachary whose moral support over the last five years has kept me on this
educational drive. For this second edition, there were fellow residents that gave me input and new insight that
has helped to improve this edition over the first.
To my parents Joseph and Leila, educators of true dedicated quality, and to whom I owe my
homeschooling education and self-motivation. Lastly to my wife Roxanne, whose patience with my
ambitiousness
biti kknows no bboundaries.
d i
Thank you All, ©™
Chaim September 9, 2008
How to use this Flashcard review
• These cards are intended to cover most of the aspects of the
Neurosurgeryg y Board Examination. They y are not a COMPLETE
review and therefore they are not intended to replace textbooks. We
would advise using these cards during the last couple of weeks
before your board exam except for the pathology section which you
should go through all year to better remember the photographs in it
((heavilyy encountered during
g the boards!).
) BOARD FAVORITE
questions are of extreme importance and most likely to bump into
during the boards, so make you sure you know how to answer them
right.
• Good luck!
• Chaim B.B Colen,
Colen M.D.,
M D Ph
Ph.D.
D
©™
Faculty Reviewers
Murali Guthikonda, MD
Professor and Chairman
D
Department
t t off Neurological
N l i lS Surgery
Wayne State University School of Medicine
Detroit, Michigan

Setti Rengachary, MD
Associate Chairman
D
Department
t t off Neurological
N l i lS Surgery
Wayne State University School of Medicine
Detroit, Michigan

William, J. Kupsky, MD
Department of Neuropathology
W
Wayne State
St t University
U i it School
S h l off Medicine
M di i
Detroit, Michigan ©™
Forward
• With ever increasing scope and complexity of knowledge base, the current
day trainee or practitioner of neurosurgery finds it difficult to keep up with
the explosion of neurosurgical information. This is compounded by a
healthy growth in specialization in various branches of neurosurgery.
• Chaim has made an attempt to make life simpler by incorporating small
quanta of knowledge on flashcards accompanied by clear and simple
illustrations. The user may review as few or as many cards as his/her time
will allow. Although not meant to be substitutes for standard comprehensive
t t and
texts d atlases,
tl these
th cards
d help
h l tto refresh
f h ththe information
i f ti learned
l d ffrom
the bedside, operating room and standard books. Each card represents a
mini-examination with instant access to appropriate answers.
• This is a fun way to recall neurosurgical information especially before an
upcoming test.

©™
Setti S. Rengachary, M.D.
Department of Neurological Surgery
Physician Contributing Authors

Rivka R. Colen,, MD Mahmoud Rayes, MD


Department of Radiology Department of Neurological
The Massachusetts General Hospital Surgery
Harvard Medical School WSU School of Medicine
Boston, Massachusetts
Erika Peterson, MD
Doha Itani,
Itani MD UT Southwestern,,
Department of Pathology Department of Neurological
Surgery
WSU School of Medicine
Dallas, Texas
Detroit, Michigan

©™
Contributing Medical Students
Darmafall, Kristyn Galinato, Anthony Larson, Sarah
Wayne State University Wayne State University Wayne State University
School of Medicine School of Medicine School of Medicine
Class of 2012 Class of 2012 Class of 2012

Davis, Naomi Gotlib, Dorothy Martinez, Derek


Wayne State University Wayne State University Wayne State University
School of Medicine School of Medicine School of Medicine
Class of 2011 Class of 2009 Class of 2011

Dub, Larissa
Dub Kozma, Bonita
Kozma Matthew Smith
Wayne State University Wayne State University Wayne State University
School of Medicine School of Medicine School of Medicine
Class of 2012 Class of 2008 Class of 2011

Faulkiner, Rodney Lai, Christopher Matto, Shereen


Wayne State University Wayne State University Wayne State University
School of Medicine School of Medicine School of Medicine
Class of 2012 Class of 2010 Class of 2012
©™
Contributing Undergraduates
Jeffrey P. Kallas
Wayne State University
Class of 2010

Abhinav Krishnan
Wayne State University
Class of 2010

Peter Paximadis
Wayne State University
Class of 2008

©™
Q? Neuropathology
• This cerebellar specimen is
consistent with:
A. Lewy body
B. Neuritic plaque
C. Kuru plaque
D. Hirano body

©™

2
A Neuropathology
A. Neuropathology
Section
• The correct answer is C, Kuru plaque. Kuru Plaque
• Pathology of variant Creutzfeldt-Jakob
disease – Kuru demonstrating a “kuru
plaque” in cerebellum.
• Accumulation of prion protein PrP in the
form of amyloid plaques occurs in some
forms of pprion disease.
• Hirano bodies (rod-shaped eosin
inclusions) are seen in Alzheimer’s
disease.

PAS
Cerebellum: ©™
Liberski PP. Amyloid plaques in transmissible spongiform encephalopathies (prion diseases). Folia
Neuropathol. 2004;42 Suppl B:109-19. Review. granular cell layer

Cerebellar Infection, Prion, Creutzfeldt-


Jakob disease,
disease Kuru,
Kuru amyloid
Q? Neuropathology
• This brain specimen is most
consistent with:
A. Lewy body
B. Neuritic plaque
C. Kuru plaque
D. Hirano body

©™

3
A
A. Neuropathology BOARD FAVORITE!

• The correct answer is D, Hirano body. Hirano body


• Hirano bodies (rod-shaped eosin
inclusions of mostly actin filaments) are
seen in Alzheimer’s disease.
• Accumulation of prion protein PrP in
the form of amyloid plaques occurs in
the ppathology
gy of variant Creutzfeldt-
Jakob disease – Kuru demonstrates
“kuru plaques” in the cerebellum.
• Lewy bodies are seen in Parkinson’s
disease and diffuse Lewy body
Section of hippocampus
disease These are formed from α-
disease.
synuclein. ©™
Kuljis RO: Modular corticocerebral pathology in Alzheimer's disease. In: Mangone CA, Allegri RF, Ariza, eds. Dementia: A Multidisciplinary
Approach. 1997: 143-55.

Degenerative disease, Alzheimer’s


disease Hirano bodies
disease,
Q? Neuropathology
• What is the most likely diagnosis seen
in this fundoscopic examination?
A. Drusen bodies
B. Retinopathy of diabetes
C. Retinal hemorrhage
D. Pseudopapilledema

Used with permission from Handbook of Neurosurgery and


Neurology in Pediatrics; By Chaim B. Colen, MD, PhD.

©™

9
A
A. Neuropathology
• The correct answer is C, retinal hemorrhages.
• Shaken baby syndrome was described by Caffey as a clinical triad: subdural
hematomas, subarachnoid hemorrhages, and retinal hemorrhages.
• "Shaken baby syndrome" (SBS) results in intracranial and intraocular hemorrhages
with no evidence of external trauma. The cause of these injuries is vigorous shaking
of an infant being held by the chest, shoulders, or extremities.
• Physicians are mandated to report cases of child abuse,
abuse elder abuse,
abuse sexual abuse
abuse,
domestic violence, and assault. Physicians are also required to notify the authorities if
anyone has a specific plan to commit suicide or homicide.
• Drusen bodies aka “pseudopapilledema” are colloid or hyaline bodies lying beneath
the retinal pigment epithelium. They may occur either secondary to changes in the
choroid that affect the pigment epithelium or as an autosomal dominant disorder of
the retinal pigment epithelium. ©™
Martin HA, Woodson A, Christian CW, Helfaer MA, Raghupathi R, Huh JW. Shaken baby syndrome. Crit Care Nurs Clin North Am. 2006 Sep;18(3):279-86. Review.

Shaken baby syndrome


syndrome, Pediatrics
Pediatrics, Retinal
hemorrhages, subdural hematomas,
subarachnoid hemorrhages
Q? Neuropathology
• Which pairing is most accurate for the localization of medulloblastomas in the pediatric
and adult population?
A. Children : Midline, Adults : Midline
B. Children : Lateral, Adults : Lateral
C. Children : Lateral, Adults : Midline
D. Children : Midline, Adults : Lateral

©™

54
A
A. Neuropathology BOARD FAVORITE!

• The correct answer is D, children :


midline, adults: lateral

Homer-Wright rosette –(pseudorosette)


radially arranged nuclei surrounding central
fib ill
fibrillary material
t i l ((red
d arrow);
) hi
histopathological
t th l i l MRI bbrain
i -contrast
t t enhanced
h d
features of medulloblastoma. sagittal image showing the midline ©™
location of the tumor in a pediatric
patient.

Neoplasm,
p Cerebellum, PNET, medulloblastoma,
location of medulloblastoma, Homer-Wright
rosette
Q? Neuropathology
• Compared to pilocytic
astrocytoma:
A. This tumor has a better
prognosis
B. This tumor has a similar
prognosis.
C. This tumor has a worse
prognosis
D. Prognosis is irrelevant

©™

114
A
A. Neuropathology BOARD FAVORITE!

• The correct answer is C, this tumor has a worse prognosis..


• Gemistocytic astrocytoma is WHO grade II and is distinctive in its appearance, having
astrocytes
t t withith generous eosinophilic
i hili cytoplasm
t l with
ith a ““stuffed”
t ff d” stellate
t ll t appearance.
• Low-grade astrocytomas with a significant fraction of gemistocytes progress more rapidly
and typically carry a p53 mutation.
Secondary GBM
Gemistocytic astrocytoma

PDGFR increase
Grade II Grade III
Progenitor cell astro astro Secondary GBM
P53 loss

pRB loss,
p oss, NK4α
α increase,
c ease, PTEN,, loss
oss
©™
Krouwer HG, Davis RL, Silver P, Prados M. Gemistocytic astrocytomas: a reappraisal. J Neurosurg. 1991 Mar;74(3):399-406.

neoplasm, astrocytoma, gemistocytic


astrocytoma, WHO grade III
Q? Neuropathology
• This muscle specimen
(Gomori stain) is consistent
with
ith which
hi h di
disorder?
d ?
A. Myoclonic seizures
B. Disuse atrophy
C. Steroid atrophy
D Denervation atrophy
D.

©™

138
A
A. Neuropathology BOARD FAVORITE!

• The correct answer is A, myoclonic


seizures.
• Mitochondrial disorders (e.g.
(e g
MELAS, MERRF) clinically are
associated with seizures and muscle
biopsy shows ragged-red fibers and
rimmed vacuoles.
• Type 1 fiber atrophy- think muscle
di
disease! ! -myotonic
t i ddystrophy
t h andd
congenital myopathy
• Type 2 muscle atrophy- think
neurogenic or muscle disease! –
denervation, disuse, myasthenia RAGGED RED FIBERS RIMMED VACUOLE
gravis, steroids and paraneoplastic
syndrome (BOARD FAVORITE!)
Fukuhara N, Tokiguchi S, Shirakawa K, Tsubaki T. Myoclonus epilepsy associated with ragged-red fibres (mitochondrial abnormalities ): disease entity or a
©™
syndrome? Light-and electron-microscopic studies of two cases and review of literature. J Neurol Sci. 1980 Jul;47(1):117-33.

Muscle, Mitochondrial disease, seizures, ragged-


red fibers, rimmed vacuoles, type 1 fiber atrophy,
type 2 fiber atrophy
Q? Neuropathology
• This peripheral nerve biopsy is
consistent with:
A Myasthenia
A. M asthenia gra
gravis
is
B. Onion bulb formation
C. Wallerian degeneration
D. Neurofibroma

©™

139
A
A. Neuropathology BOARD FAVORITE!

• The correct answer is B, onion bulb


formation.
• Onion bulb formation occurs
after repeated cycles of
demyelination and remyelination
result in a thick layer of abnormal
myelin around the peripheral axons.
These changes cause what is
referred to as an onion bulb
appearance.
• May be seen in a few disorders:
Charcot-Marie-Tooth (CMT) and
Refsum’s disease (phytanic acid
disorder). ONION BULB (electron
ONION BULBS microscopy)
Bornemann A, Hansen FJ, Schmalbruch H: Nerve and muscle biopsy in a case of hereditary motor and sensory neuropathy type III with basal lamina onion
©™
bulbs. Neuropathol Appl Neurobiol 1996 Feb; 22(1): 77-81.

Peripheral nerve, Onion bulb formation, myelin,


Charcot-Marie-Tooth, Refsum’s disease
Q? Neuropathology
• Symptoms of a patient with this
diagnosis might be alleviated with
which
hi h off the
th following
f ll i ddrugs:
A. Retroviral drug
B. Anticholinesterase inhibitor
C. Beta-interferon
D Trimethoprim-
D. Trimethoprim
sulfamethoxazole

©™

140
A
A. Neuropathology
• The correct answer is A,
anticholinesterase inhibitor.
• Mi
Microscopy shows
h an amyloid
l id A l id plaque
Amyloid l
plaque and neurofibrillary tangles
that would be consistent with the
diagnosis of Alzheimer’s disease.
Neurofibrillary tangle
• Donepezil is a synthetic
noncovalent reversible inhibitor
of acetylcholinesterase (AChE)
for the treatment of mild to
moderate dementia associated with
Alzheimer's disease.
• Other drugs include galantamine
and rivastigmine. ©™
Kuljis RO: Modular corticocerebral pathology in Alzheimer's disease. In: Mangone CA, Allegri RF, Ariza, eds. Dementia: A Multidisciplinary Approach. 1997: 143-
55.

Degenerative disease, Dementia, Alzheimer’s


disease, neurofibrillary tangles, amyloid plaque,
Donepezil, Acetylcholinesterase inhibitor
Q? Neuropathology
• This brain specimen is most
consistent with:
A. Alzheimer’s disease
B. Parkinson’s disease
C. Pick’s disease
D. Wilson’s disease

©™

142
A
A. Neuropathology BOARD FAVORITE!

• The correct answer is B,


Parkinson’s disease.
• Microscopy shows melanophages
and neuronal cell loss in the
substantia nigra pars compacta.
• Lewy bodies are also seen.
These are formed from α-
synuclein.
• Pick bodies are seen in Pick’s
disease using silver stain.
NEURONAL LOSS LEWY BODY

Hodaie M, Neimat JS, Lozano AM. The dopaminergic nigrostriatal system and Parkinson's disease: molecular events in development, disease, and cell death, and
©™
new therapeutic strategies. Neurosurgery. 2007 Jan;60(1):17-28; discussion 28-30.

Degenerative disease, Movement disorder,


Parkinson’s disease, melanophages, lewy bodies,
alpha-synuclein
Q? Neuropathology
• This neonatal brain specimen is
consistent with which hemorrhage grade?
A. Grade I
B. Grade II
C. Grade III
D. Grade IV

©™

148
A
A. Neuropathology BOARD FAVORITE!

• The correct answer is C, grade III.


• This gross specimen demonstrates germinal matrix IVH and hydrocephalus
hemorrhage (GMH) and intraventricular hemorrhage
(IVH) and hydrocephalus which are the most
common and most important neurologic injuries in GMH
preterm neonates.
• Premature infants lack the ability to autoregulate the
cerebral blood pressure. Fluctuations in cerebral
bl d pressure andd flow
blood fl can rupturet th the primitive
i iti
germinal matrix vessels or lead to infarction of the
metabolically active germinal matrix. Neurologic
sequelae, include cerebral palsy, mental retardation,
and seizures. GRADE 3
Grading System
Grade 1 - confined GMH
Grade 2 - IVH without hydrocephalus. Levy ML, Masri LS, McComb JG: Outcome for preterm infants with germinal ©™
Grade 3 - IVH with associated hydrocephalus matrix hemorrhage and progressive hydrocephalus. Neurosurgery 1997 Nov;
Grade 4 - Intraparenchymal hemorrhage 41(5): 1111-7; discussion 1117-8.

Vasculature, hemorrhage, hemorrhage grading,


germinal matrix hemorrhage
hemorrhage, intraventricular
hemorrhage, Pediatrics
Q? Neuropathology
• This pathology is most likely due
to:
A. Vitamin A toxicity
B. Cyanocobalamine
deficiency
C. Thiamine deficiency
D Vitamin B6 deficiency
D.
E. Alcohol intake

©™

149
A
A. Neuropathology BOARD FAVORITE!

• The correct answer is C, thiamine


deficiency.
• Thiamine deficiency is seen in chronic Old infarction
malnourished alcoholics. Note the old
infarction that has occurred in the
mamillary bodies.
• Wernicke-Korsakoff’s syndrome
y –
subacute amnesia seen in damage to
the mamillary bodies and may be
caused by thiamine deficiency and
may be reversible with the
administration of thiamine.
©™
Torvik A. Topographic distribution and severity of brain lesions in Wernicke's encephalopathy. Clin Neuropathol. 1987 Jan-Feb;6.

Nutritional deficiency, Thiamine deficiency,


Wernicke-Korsakoff syndrome, mamillary bodies
Q? Neuropathology
• This brain specimen is most
consistent with which disease?
A. Wilson’s (hepatolenticular
degeneration)
B. Alzheimer’s
C. Pick’s
D Parkinson
D. Parkinson’ss
E. Huntington’s

©™

151
A
A. Neuropathology BOARD FAVORITE!

• The correct answer is A, Wilson's disease


(hepatolenticular degeneration).
• Wilson’ss disease is caused by accumulation
Wilson
of copper in the brain with decreased serum
copper and ceruloplasmin and increased
urine copper. Kayser-Fleischer rings form
around the iris.
• Microscopy demonstrates Alzheimer type 2
astrocytes
t t which
hi h hhave a llarge clear
l vesicular
i l
nuclei and prominent nucleoli. Proliferation of
large protoplasmic astrocytes such as
Opalski cells may be seen.
• Research suggests that these reactive
astrocytes may be involved in the process of
copper detoxification in WD.
Bertrand E, Lewandowska E, Szpak GM, Hoogenraad T, Blaauwgers HG, Czlonkowska A, Dymecki J. Neuropathological analysis of pathological forms of
©™
astroglia in Wilson's disease. Folia Neuropathol. 2001;39(2):73-9.

Degenerative disease, Wilson’s disease,


Alzheimer type 2 astrocytes, Opalski cells,
Copper
Q? Neuropathology
• This muscle biopsy specimen is
consistent with:
A. Congenital myopathy
B. Myotonic dystrophy
C. Steroid myopathy
D. Mitochondrial myopathy
1
Note: Type 1 fibers labeled (1).

©™

152
A
A. Neuropathology BOARD FAVORITE!

• The correct answer is C, steroid myopathy.


• Steroid myopathy is characterized by
type 2 myofiber atrophy!

TYPE 2 ATROPHY

• Type 1 fiber atrophy- think muscle


disease! -myotonic dystrophy and
congenital
g myopathy
y p y
• Type 2 muscle atrophy- think neurogenic
or muscle disease! –denervation, disuse,
myasthenia gravis, steroids and
STEROID MYOPATHY
paraneoplastic syndrome (BOARD
FAVORITE)

©™
Griggs RC, Mendell JR, Miller RG: Congenital myopathies. In: Evaluation and Treatment of Myopathies. Philadelphia: FA Davis Co; 1995: 211-46.

Muscle, myopathy, steroid myopathy, type 2 fiber


atrophy
Q? Neuropathology
• This gross brain specimen is most
consistent with:
A. Adrenoleukodystrophy
B. Multiple sclerosis
C. Mitochondrial myopathy
D. Canavan’s disease

©™

154
A
A. Neuropathology BOARD FAVORITE!

Periventricular
• The correct answer is B, multiple sclerosis.
location
• Multiple sclerosis (MS) is an idiopathic
(autoimmune?) inflammatory demyelinating
disease of the CNS. Animal model
experimental allergic encephalomyelitis
demonstrated by autoimmunity to myelin
basic proteins. (BOARD FAVORITE)
• Episodes of de/remyelination result in a
chronic
h i bburned-out
d t plaque
l with
ith relative
l ti
preservation of axons and gliosis (recently
axon transection has been reported in acute
exacerbations). Chronic plaques
• Incidence is higher in Caucasians. Female-
to-male ratio is 2:1.
• Classic presentation -optic neuritis, ©™
transverse myelitis, internuclear Noseworthy JH, Lucchinetti C, Rodriguez M, Weinshenker BG: Multiple sclerosis. N Engl J
Med 2000 Sep 28; 343(13): 938-52.
ophthalmoplegia, paresthesias

Demyelinating
y g disease, Inflammatoryy disease,
Multiple Sclerosis, myelin basic protein
Q? Neuropathology
• The gross spinal cord specimen (A)
is most consistent with:
A. Adrenoleukodystrophy
B. Multiple sclerosis
C. Amyotrophic lateral sclerosis
D. Neurofibromatosis

A B

©™

156
A
A. Neuropathology BOARD FAVORITE!

Ventral nerve Normal


• The correct answer is C, amyotrophic
root atrophy ventral roots
lateral sclerosis.
• Amyotrophic lateral sclerosis
progressive loss of upper and lower motor
neurons.
• Pathology: progressive loss of anterior
horn cells leads to atrophy of ventral
nerve fibers and of skeletal muscles;
i lloss off primary
progressive i motor
t neurons
in motor cortex leads to degeneration of
corticospinal/ corticobulbar tracts.
• Adrenoleukodystrophy (ALD)-
leukodystrophy is an x-linked recessive A B
disease due to deficiency in peroxisomal
lipid oxidation.
©™
Gartner J, Braun A, Holzinger A , et al: Clinical and genetic aspects of X-linked adrenoleukodystrophy. Neuropediatrics 1998 Feb; 29(1): 3-13.

spinal cord, Degenerative disease, Amyotrophic


lateral sclerosis, Upper motor neuron, Lower
motor neuron
Q? Neuropathology
• This gross brain specimen shows
atrophy that is consistent with:
A. Multiple sclerosis
B. Pick’s disease
C. Alzheimer’s disease
D. Acute cerebral infarction

©™

157
A
A. Neuropathology BOARD FAVORITE!

Frontal Atrophy
• The correct answer is Alzheimer’s
disease.
• Al h i ’ disease
Alzheimer’s di i th
is the mostt
common dementing illness in adults,
characterized by progressive
dementia over several years. There
is increased frequency with
increasing age and in familial cases,
cases
usually earlier onset.
• Note the diffuse brain atrophy of
Alzheimer's disease, unlike Pick’s
disease which has mostly frontal Temporal Atrophy
lobe atrophy.
p y
• Risk is increased in Down’s ©™
syndrome (BOARD FAVORITE). Kuljis RO: Modular corticocerebral pathology in Alzheimer's disease. In: Mangone CA,
Allegri RF, Ariza, eds. Dementia: A Multidisciplinary Approach. 1997: 143-55.

Degenerative disease, Alzheimer’s disease,


dementia, diffuse atrophy
Q? Neuropathology
• The pathology seen here is most likely due to:
A. Cerebellar astrocytoma
B. Alcoholism
C. Trauma
D. Ruptured aneurysm
E. Arteriovenous malformation

©™

158
A
A. Neuropathology BOARD FAVORITE!

• The correct answer is….


• To obtain the answer to this question and to
view over 400 more comprehensive
pathology questions please purchase the
full product here!

©™
Bolla L, Palmer RM: Paraneoplastic cerebellar degeneration. Case report and literature review. Arch Intern Med 1997 Jun 9; 157(11): 1258-62.

Degeneration, Cerebellum, Alcohol related


degeneration, superior vermis of cerebellum
Neurobehavioral Section
Neurobehavioral Section

©™
Q? Neurobehavioral
• Which of the following types of mental retardation is associated with a maternally-
inherited deletion of segment 11-13 of chromosome 15?
A. Prader-Willi syndrome
B. Rett’s syndrome
C. Fragile X syndrome
D. Angelman’s syndrome

©™

2
A
A. Neurobehavioral BOARD FAVORITE!

• The correct answer is D, Angelman’s syndrome.


• Angelman’s syndrome is classically associated with a maternally-inherited deletion of
segment 11-13
11 13 on chromosome 15. 15 This ssyndrome
ndrome is characterized
characteri ed by
b mental retardation,
retardation
abnormal gait, speech impairment, seizures, and inappropriate happy behavior that is often
described as “happy puppet syndrome.”
• Prader-Willi syndrome consists of the same deletion (segment 11-13 of chromosome 15) but is
located on the paternal chromosome 15. (Remember: P-P, Paternal-Prader-Willi)
• Rett’s syndrome is a progressive neurodevelopmental disorder that affects females only.
only The
cause of Rett’s syndrome is the MeCP2 gene, located on the long arm of the X chromosome.
• Fragile X syndrome is the most common form of inherited mental retardation. It is caused by
a CGG repeat on the X chromosome, and effects include long face, enlarged ears, and macro-
orchidism.

©™

Fauci A, Braunwald E, et al. Harrison’s Principles of Internal Medicine 17th Edition. New York, NY: McGraw-Hill, Medical Pub. Division
2005. p. 407-14.

Classification: Neurobehavioral &


Psychiatry,
y y, mental retardation,, ggenetics
Q? Neurobehavioral
• Fragile X syndrome is the most common form of inherited mental retardation. Somatic
abnormalities seen commonly in Fragile X patients include a long face, large ears, and
macroorchidism Which of the following genetic abnormalities is the most likely cause of Fragile
macroorchidism.
X syndrome?
A. Absence of segment 11-13 on chromosome 5
B. MeCP2 gene on the long arm of the X chromosome
C. CGG trinucleotide expansion of an area of the X chromosome
D Inheritance
D. I h i off an extra copy off chromosome
h 21

©™

3
A
A. Neurobehavioral BOARD FAVORITE!

• The correct answer is C, CGG trinucleotide expansion of an area of the X chromosome.


• Fragile X syndrome is the most common form of inherited mental retardation. It is caused
b a CGG repeat on the X chromosome
by chromosome, and effects incl
include
de long face
face, enlarged ears
ears, and
macro-orchidism. It affects males more than females.
• Angelman’s syndrome is classically associated with a maternally-inherited deletion of
segment 11-13 on chromosome 15. This syndrome is characterized by mental retardation,
abnormal gait, speech impairment, seizures, and inappropriate happy behavior that is often
described as “happy
happy puppet syndrome.
syndrome ”
• Prader-Willi syndrome consists of the same deletion (segment 11-13 of chromosome 15) but
is located on the paternal chromosome 15.
• Rett’s syndrome is a progressive neurodevelopmental disorder that affects females only. The
cause of Rett’s syndrome is the MeCP2 gene, located on the long arm of the X chromosome.

©™
Sadock B, Sadock V. Kaplan & Sadock’s Synopsis of Psychiatry, 10th Edition. Philadelphia, PA: Lippincott Williams & Wilkins 2007.
p. 1143.
Q? Neurobehavioral
• A 6 year-old boy has been sent for evaluation by a psychiatrist for attention deficit
hyperactivity disorder (ADHD) because he has not been completing his homework and is
constantly
t tl interrupting
i t ti his
hi teacher
t h andd classmates.
l t His
Hi teacher
t h reports t that
th t hhe hhas
trouble sitting still. Upon questioning his parents, they report that their son’s inattention
and hyperactivity started over 6 months ago. With a diagnosis of attention deficit
hyperactivity disorder in this child, which of the following would be the first-line
treatment?
A. Individual psychotherapy
B. Selective serotonin reuptake inhibitors & tricyclic antidepressants
C. Methylphenidate (Ritalin)
D. Monoamine oxidase inhibitor (MAOI)

©™

4
A
A. Neurobehavioral BOARD FAVORITE!

• The correct answer is C, methylphenidate (Ritalin).


• Methylphenidate is considered the first line therapy for children diagnosed with
attention deficit hyperactivity disorder (ADHD). Significant improvement is seen in
roughly 75% of children when taking methylphenidate. Individual psychotherapy
with a focus on behavioral modification can be used as adjunctive therapy, but is not
considered first-line. Selective serotonin reuptake inhibitor (SSRI) and tricyclic
antidepressant (TCA) therapy can be used as adjunctive pharmacologic therapy for
ADHD, but is not a first-line treatment. Monoamine oxidase inhibitors (MAOIs) are also
antidepressants, but are not used in the treatment of attention deficit hyperactivity
disorder. Of note, it is important to recognize that in order for a diagnosis of attention
deficit hyperactivity disorder to be made, symptoms must be present for a minimum of
6 months, and the child must have the onset of symptoms before the age of 7.
©™
Sadock B, Sadock V. Kaplan & Sadock’s Synopsis of Psychiatry, 10th Edition. Philadelphia, PA: Lippincott Williams & Wilkins 2007. p.
1211-14.
Q? Neurobehavioral
• The treatment of attention deficit hyperactivity disorder (ADHD) classically involves the
use of stimulants, such as methylphenidate (Ritalin). Which of the following are common
adverse
d effects
ff t off methylphenidate?
th l h id t ?
A. Tachycardia, insomnia, and abdominal pain
B. Bradycardia and hyperphagia
C. Suicidal thoughts, gestures, or ideation
D Increased intracranial pressure (ICP)
D.

©™

5
A
A. Neurobehavioral BOARD FAVORITE!

• The correct answer is A, tachycardia, insomnia, and abdominal pain.


• Tachycardia, insomnia, and abdominal pain are all reported side effects of
methylphenidate.
• Bradycardia is not seen. Methylphenidate is also reported to cause a decrease in
appetite, not hyperphagia.
• Increased intracranial pressure is not a side effect of methylphenidate use.

©™
Sadock B, Sadock V. Kaplan & Sadock’s Synopsis of Psychiatry, 10th Edition. Philadelphia, PA: Lippincott Williams & Wilkins 2007. p.
1211-12.
Q? Neurobehavioral
• At which age are developmental milestones of group play, ability to ride a tricycle, and
the ability to copy a line or circle drawing present?
A. 2 years
B. 3 years
C. 4 years
D. 5 years

©™

7
A
A. Neurobehavioral BOARD FAVORITE!

• The correct answer is B, 3 years.


• The above milestones are classically present at age 3.
• 4 year-olds, on the other hand, can participate in cooperative play and construct simple
drawings (stick figure drawings, for example).
• Milestones like abstract reasoning and the formation of personality occur during
adolescence (age 11 for girls and age 13 for boys).

©™
Bhushan V, Le T, Chandwani R, Ozturk A. First Aid for the USMLE Step 1 2005: A Student to Student Guide. New York, NY:
McGraw-Hill 2005. p. 120.
Q? Neurobehavioral
• A 25 year-old medical student is arrested at school after stealing a laboratory scale. He
has a history of multiple childhood arrests, both for abusing his neighbor’s dog and
setting
tti fires
fi tot hhouses nearby.
b HiHis school
h l records
d showh a hihistory
t off disciplinary
di i li action
ti
for aggressiveness and fighting with classmates. During questioning, he shows no
remorse for his actions. Prior to the age of 18, this individual would most probably
have been diagnosed with which of the following?
A. Antisocial personality disorder
B. Schizotypal personality disorder
C. Conduct disorder
D. Borderline personality disorder
E. Histrionic personality disorder
©™

9
A
A. Neurobehavioral BOARD FAVORITE!

• The correct answer is C, conduct disorder.


• Prior to age 18, individuals with the above behavior are classified as having conduct
disorder. Currently, as a 25 year-old medical student, he would be classified as
having antisocial personality disorder.
• Borderline personality disorder is characterized by unstable moods and labile,
unstable relationships.
• Histrionic personality disorder is characterized by excessive attention-seeking
behavior and emotionality. These individuals are dramatic, sexually provocative,
and unable to form long-lasting relationships.
• Schizotypal personality disorder is characterized by individuals with eccentric
behavior and peculiar thought patterns. They commonly exhibit odd beliefs or
magical thinking.
thinking
©™
Sadock B, Sadock V. Kaplan & Sadock’s Synopsis of Psychiatry, 10th Edition. Philadelphia, PA: Lippincott Williams & Wilkins 2007.
p. 1220-1224.
Q? Neurobehavioral
• Which of the following is a metabolic abnormality associated with anorexia nervosa?
A. Hypochloremic hyperkalemic metabolic alkalosis
B. Hypochloremic hypokalemic metabolic alkalosis
C. Hyperkalemic metabolic acidosis
D. Hyponatremia

©™

11
A
A. Neurobehavioral BOARD FAVORITE!

• The correct answer is B, hypochloremic hypokalemic metabolic alkalosis.


• Anorexia nervosa is an eating disorder in which body weight is 15% below normal.
Additionally, there is an intense fear of gaining weight, disturbed body image, and
amenorrhea.
• The most common metabolic complication resulting from anorexia nervosa is
hypochloremic hyperkalemic metabolic alkalosis. Lanugo, or fine body hair, is also
commonlyy seen in the anorexic ppatient.
• Hypochloremic hypokalemic metabolic alkalosis is commonly seen in bulimia nervosa
(secondary to emesis), not in anorexia.
• Hyponatremia and hyperkalemic acidosis are usually not seen in anorexia nervosa.

©™
Sadock B, Sadock V. Kaplan & Sadock’s Synopsis of Psychiatry, 10th Edition. Philadelphia, PA: Lippincott Williams & Wilkins 2007.
p. 731-2.
Q? Neurobehavioral
• A 23 year-old female is brought in to your psychiatric clinic by her parents for an eating
disorder evaluation. Her parents report that they rarely see her eating and are
concernedd th
thatt she
h iis suffering
ff i ffrom anorexia
i nervosa. Y You perform
f a hi
history
t andd
physical examination on the patient without her parents present, and find that she is
mildly overweight. You notice that she has calloused knuckles on her right hand.
When you ask her questions about her body image, she informs you that she is
extremely distressed by her inability to achieve a lower body weight. What is the most
lik l di
likely diagnosis?
i ?
A. Anorexia nervosa
B. Bulimia nervosa, purging type
C. Bulimia nervosa, non-purging type
D This patient is not suffering from an eating disorder.
D. disorder
©™

12
A
A. Neurobehavioral BOARD FAVORITE!

• The correct answer is B, bulimia nervosa, purging type.


• This patient is suffering from bulimia nervosa, purging type.
• C
Common findings
fi di iin a bulimic
b li i patient
ti t off the
th purging
i ttype include
i l d hypochloremic
h hl i hhypokalemic
k l i
metabolic alkalosis, dental erosion, and calloused knuckles (all as a result of self-induced
vomiting).
• Interestingly, while anorexia nervosa patients have body weight 15% below-average, bulimia nervosa
patients often have normal body weights or are slightly overweight.
• In addition,, bulimia is described as ego-dystonic
g y ((distressingg to the ppatient).
) These ppatients are aware
that they have a problem and are distressed by it, making them more likely to seek help than anorexic
patients. Anorexia nervosa, on the other hand, is ego-syntonic (not distressing to the patient) and
patients suffering from anorexia are less likely to seek help.
• The purging type of bulimia nervosa involves vomiting, the use of laxatives, or the use of diuretics as a
means to lose weight. The non-purging type of bulimia involves excessive exercise or fasting. Note
that while anorexia and bulimia can potentially involve excessive fasting,
fasting the anorexic will have a low
body weight while the bulimic will have a normal body weight or be overweight. ©™
Sadock B, Sadock V. Kaplan & Sadock’s Synopsis of Psychiatry, 10th Edition. Philadelphia, PA: Lippincott Williams & Wilkins 2007. p.
735-8.
Q? Neurobehavioral
• A 23 year-old female gives birth to a healthy baby girl. However, on further inspection
the newborn is noted to be deaf, has congenital cataracts, and has cardiac
abnormalities
b liti with
ith possible
ibl mental
t l retardation.
t d ti The
Th mother
th reportedt d having
h i an
unremarkable pregnancy except during the first trimester when she reported having a
mild illness consisting of a maculopapular rash, arthritis, aching joints, and diffuse
swollen lymph nodes lasting 2-3 days. These congenital anomalies could have best
been prevented by which of the following?
A. Treatment of penicillin prior to pregnancy
B. Receiving the measles, mumps, and rubella vaccine prior to pregnancy
C. Avoiding uncooked pork and cat litter during the pregnancy
D. Maintaining on highly active antiretroviral therapy (HAART) during the pregnancy
E Receiving
E. R i i th the varicella
i ll zoster
t vaccinei prior
i tto pregnancy
©™

21
A
A. Neurobehavioral
• The correct answer is B, receiving the measles, mumps and rubella vaccine prior to pregnancy.
• This patient who was not immunized to rubella, contracted the disease during her pregnancy as noted
by her classic symptoms and classical congenital defects.
defects Congenital rubella syndrome (CRS) has the
highest transference rate during the first trimester.
• Choice A , refers to syphilis which usually manifests as a late abortion or congenital syphilis with eighth
nerve deafness, saber shins, Hutchinson teeth, and a saddle nose
• Choice C, refers to Toxoplasmosis gondii a common protozoan parasite whose transmission also
occurs during the first trimester. However, this parasite often manifests with fevers, seizures,
chorioretinitis hepatosplenomegaly
chorioretinitis, hepatosplenomegaly, jaundice,
jaundice and hydrocephaly or microcephaly.
microcephaly
• Choice D, refers to HIV infected mothers. Transmission is believed to occur late in pregnancy or during
delivery.
• Choice E, refers to the varicella zoster virus , which can also occur during the first trimester, but in this
trimester the effects are often a spontaneous abortion and possible teratogenic effects but not the series
of effects listed.

©™
Charles R. B. Beckmann et al. Obstetrics and Gynecology Fifth Edition. Philadelphia, PA: Lippincott Williams & Wilkins 2006. p. 181- 184.
Q? Neurobehavioral
• A 19 year old women presents to the psychiatrist with her mother. The patient reports having
heard voices for the past month. On further questioning the psychiatrist discovers that for the
past 2 months the patient has had grossly disorganized behavior and a delusion that someone
is following her. Her mother states that she was completely normal until two months ago.
What is the most likely diagnosis at this time?
A. Schizophrenia
B. Brief psychotic disorder
C Schizoaffective
C. S hi ff ti di disorder
d
D. Schizophreniform
E. Schizotypical

©™

31
A
A. Neurobehavioral BOARD FAVORITE!

• The correct answer is D, schizophreniform.


• While this patient displays the DSM IV criteria for schizophrenia, her symptoms have only
been present for the past 2 months. The DSM IV requires that symptoms be present for
the past 6 months in order to diagnose schizophrenia (choice A). If the symptoms last
under 1 month then the diagnosis of a brief psychotic disorder is made (choice B).
• Choice C, schizoaffective disorder is diagnosed when a patient meets both the DSM IV
criteria for schizophrenia
p and a mood disorder.
• Choice E, schizotypical is a personality disorder where a patient displays odd or magical
beliefs, lack of friends, and the criteria for true psychosis is not met.

Benjamin James Sadock, M.D. and Virginia Alcott Sadock, M.D. Kaplan & Sadock’s Synopsis of Psychiatry. Philadelphia, PA: Lippincott Williams & Wilkins 2003.
©™
p. 471-511; 520-529; 806-807.
Q? Neurobehavioral
• A nine year-old boy was adopted when he was one years old. His adopted papers
showed that his mother was a recovering alcoholic and admitted to drinking heavily
d i her
during h pregnancy. Since
Si he
h started
t t d school
h l att age 6,
6 he
h has
h bbeen iin special
i l
education due to learning disabilities and an IQ test of 63. According to his IQ
measurements, the boy would be classified as having:
A. Borderline intellectual functioning
B. Mild mental retardation
C. Moderate mental retardation
D. Severe mental retardation

©™

47
A
A. Neurobehavioral BOARD FAVORITE!

• The correct answer is B, mild mental retardation.


• In mild mental retardation, IQ ranges from 50/55 to approximately 70.
• In moderate mental retardation, IQ ranges from 35/40 to 50/55
• In severe mental retardation, IQ ranges from 20/25 to 35/40.
• Borderline intellectual function is a term no longer used.

©™
Shaywitz SE, Cohen DJ, Shaywitz BA. Behavior and learning difficulties in children of normal intelligence born to alcoholic mothers. J 
Pediatr. Jun 1980;96(6):978‐82
Q? Neurobehavioral
• A 45 year-old man is brought into the doctor’s office by his wife. She states that her
husband has been acting “weird” for the past few months. He does not eat food on the
l ft side
left id off hi
his plate,
l t ddoes nott shave
h th
the lleftft side
id off hi
his fface, andd will
ill deny
d that
th t hhe even
has a left arm. When the doctor asked the patient to draw a clock, he only drew a right
sided clock. Shown here. What is the best diagnosis for this patient?
A. Gerstmann’s syndrome
B. Agnosia
g
C. Dementia
D. Hemineglect
E. Homonymous hemianopsia

©™

64
A
A. Neurobehavioral BOARD FAVORITE!

• The correct answer is D, hemineglect.


• Neglect is lack of attention to one side, usually the left.
• Patients with severe neglect may fail to describe objects on the left, dress or shave
their left side.
• They may even deny their left arm as theirs and claim that it is the examiner’s arm.
• Patients with neglect do not have a hemianopia, or a primary motor or sensory deficit
for that side; rather,
rather they have damage to the non-dominant
non dominant (usually right)
hemisphere.
• Neglect is usually caused by lesions in the right frontal or parietal lobe

©™
Guttmann-Steinmetz S, Crowell JA. Attachment and externalizing disorders: a developmental psychopathology perspective. J Am Acad Child Adolesc
Psychiatry. Apr 2006;45(4):440-51.
Q? Neurobehavioral
• A 26 year-old primigravid female has been admitted to the hospital today due to new-
onset seizures that reportedly began five days ago. She has a history of rapid-cycling
bi l di
bipolar disorder.
d She Sh currentlytl complains
l i off ddepressive
i symptoms
t andd possible
ibl
suicidal ideation. The first year resident assigned to the case is interested in
administering carbamazepine (Tegretol) as a mood stabilizer for her bipolar disorder.
Which of the following adverse effects could carbamazepine potentially have on the
developing fetus?
A. Growth deficiency and microcephaly
B. Neonatal withdrawal syndrome
C. Spina bifida and agranulocytosis
D. Ebstein’s anomaly and other cardiac defects
E Floppy
E. Fl baby
b b syndrome
d
©™

66
A
A. Neurobehavioral BOARD FAVORITE!

• The correct answer is….


• To obtain the answer to this question and to view over 250 more comprehensive
neurobehavioral questions please purchase the full product here !

©™

Ebert M, Loosen P, Nurcombe B. Current Diagnosis & Treatment in Psychiatry. New York, NY: McGraw-Hill 2000. p. 320-1.

Classification: Neurobehavioral &


Psychiatry, pharmacology, carbamazepine,
t t
teratogenicity
i it
Q? Neuroradiology
• A 47 year-old male presents with a complaint
of pain “shooting down” his right leg. The
MRI is shown here
here. Which of the following is
the most appropriate management for this
patient?
A. Do nothing, MRI looks normal
B. Laminectomy and discectomy
C.
C Laminectomy and instrumentation of
L5-S1
D. Tethered cord release

©™

2
A
A. Neuroradiology BOARD FAVORITE!

• The correct answer is B, laminectomy and


discectomy.
• far lateral
Lumbar spine MRI shows a right far-lateral
herniated disc (T2 axial view). Note that the
disc bulge is not noted on the midline sagittal
view.
• Far-lateral herniated discs commonly impinge
on the exiting nerve root (same level) while
more central discs affect
t l di the ttransiting
ff t th iti nerve
root (level below).
• These patients generally do well with simple
laminectomy and discectomy. There is no need
to perform laminectomy and instrumentation of
L5-S1. There is no evidence of a thickened
filum or low conus suggestive of a tethered
©™
cord.
H. Richard Winn, M.D. Youman’s Neurological Surgery 5th Edition. Philadelphia,
PA: Elsevier 2004. p. 4514-7.

Classification: Neuroradiology, Herniated


Disc, Laminectomy and Discectomy
Q? Neuroradiology
• This MRI of the brain is most suggestive of:
A. Iron deposition
B. Macrocrania
C. Hypoxemia
D. Generalized atrophy
E. Butterfly glioma

©™

3
A
A. Neuroradiology BOARD FAVORITE!

• The correct answer is A, iron deposition.


• Hallervorden-Spatz syndrome - involves the
d
deposition
iti off iron
i especially
i ll iin th
the globus
l b
pallidus and the retina.
• Dystonia, rigidity and neurobehavioral changes
• MRI: Globus pallidus on T2-weighted images
• "Eye-of-the-tiger" sign
• Central region of hyperintensity
– Primary tissue insult
– Produces edema
• Surrounding hypointensity
– Region high in iron
– May be 2° process ©™

H. Richard Winn, M.D. Youman’s Neurological Surgery 5th Edition. Philadelphia, PA: Elsevier 2004. p. 2720.

Classification: Neuroradiology,
Hallervorden Spatz Syndrome,
Syndrome Iron
Deposition
Q? Neuroradiology
• Based on the AP and lateral cerebral
angiogram of the right external carotid
artery shown here,
here the most appropriate
statement to tell this patient is that:
A. The risk of bleeding is high due to
the cortical venous drainage.
B. The risk of bleeding is low due to the
cortical venous drainage.
drainage
C. If the flow has rapid drainage into a
sinus the bleeding risk is increased.
D. This is a normal cerebral angiogram.

©™

4
A
A. Neuroradiology BOARD FAVORITE!

• The correct answer is A. The risk of bleeding


is high due to the cortical venous drainage.
• This AP and lateral angiogram shows a dural
arteriovenous fistula with main feeders from
the occipital and middle meningeal arteries.
Multiple venous lakes and varices are
identified including two large lesions (these
suggest high pressure). Drainage of the dural
AVF is via cortical veins primarily into the
sagittal and transverse sinuses.
• The risk of bleeding is high not low with
cortical venous drainage.
• If the flow has rapid drainage into a sinus the
bleeding risk decreases.
©™
Borden JA, Wu JK, Shucart WA. A proposed classification for spinal and cranial dural arteriovenous fistulous malformations and Occipital artery
implications for treatment. J Neurosurg. 1995 Feb;82(2):166-79.

Classification: Neuroradiology, dural


arteriovenous fistula,
fistula cortical venous
drainage
Q? Neuroradiology
• Locate the central sulcus:
A. A
B. B
C. C
D. D
E. E A
F. F B
D C
E
F
©™

8
A
A. Neuroradiology BOARD FAVORITE!

• The correct answer is C.


• Sigmoid “Hook” or “Ducky Sign”
• Hook like configuration (B) of the posterior
Hook-like
surface of the precentral gyrus. The “hook”
or “ducky’s breast” corresponds to the
motor hand area. The “hook” is seen well
on CT (89%) and MRI (98%).
• Pars bracket sign A
• The paired pars marginalis (F) form a
“bracket” to each side of the interhemispheric
B
fissure at or behind the central sulcus (96%). D C
• A = precentral sulcus E
• D = postcentral gyrus F
• E = postcentral sulcus
©™
Colen CB, Handbook of Neurosurgery and Neurology in Pediatrics, 2006

Classification: Neuroradiology,
Neuroanatomy, Location of Central Sulcus
Q? Neuroradiology
• A 2 day old asymptomatic neonate is
transferred to your institution with the
f ll i image.
following i Which
Whi h off the
th following
f ll i is i
CORRECT regarding this pathology?
A. It rarely presents with congestive heart
failure.
B. It never ppresents in older children.
C. The embryonic correlate to this
malformation is the median
prosencephalic vein.
D. Treatment for this neonate should
include immediate embolization.
embolization
©™

11
A
A. Neuroradiology BOARD FAVORITE!

• The correct answer is C, the embryonic correlate to this


malformation is the median prosencephalic vein.
• Vein of Galen aneurysmal malformations (VGAM) probably
represent an arteriovenous fistula (AVF) in the wall of a
persistent embryonic vascular channel called the median
prosencephalic vein.
• By week 10 of normal fetal development, the median
prosencephalic vein regresses as the definitive internal cerebral
veins
i appear. A caudald l remnantt remains
i as the
th vein
i off GGalen.
l Vein of Galen aneurysmal malformation
• Neonatal presentation with congestive heart failure is frequent
with these malformations.
•Initial treatment of VGAM is conservative. Embolisation of a neonate is a high risk procedure and the child should be 
treated medically (for cardiac failure) until aged 5 or 6 months with regular outpatient assessment. Elective embolisation 
can be scheduled for this time with the aim of closing the AVS with cyanoacrylate glue. If the infant deteriorates 
( i
(seizures, failure to thrive, worsening cardiac failure, etc) treatment is performed earlier.
f il t th i i di f il t )t t ti f d li
Lasjuanias P. Vascular Diseases in Neonates, Infants and Children. New York: Springer Verlag 1997. ©™
Horowitz MB, Jungreis CA, Quisling RG, Pollock I. vein of Galen aneurysms: a review and current perspective. AJNR 1994; 15:1486-1496.
H. Richard Winn, M.D. Youman’s Neurological Surgery 5th Edition. Philadelphia, PA: Elsevier 2004. p. 3434.

Classification: Neuroradiology, Vascular


Pathology Vein of Galen Aneurysmal
Pathology,
Malformations
Q? Neuroradiology
• A five-month-old male infant presents with an
abnormal head shape. What is the most likely
di
diagnosis?
i ?
A. Sagittal synostosis
B. Bicoronal synostosis
C. Left unicoronal synostosis
D Right unicoronal synostosis
D.
E. Metopic synostosis

©™

12
A
A. Neuroradiology
• The correct answer is C, left unicoronal synostosis.
Contralateral Ipsilateral
• Coronal synostosis can occur either on the right or left side •Frontal bossing •Flattened forehead
(unicoronal) or both (bicoronal
(unicoronal), (bicoronal.)) It frequently occurs prenatally,
prenatally •Displaced fontanelle
p •Raised brow
(Harlequin Eye)
and appears to occur more commonly in males. Restriction of •Temporal bulging

normal cranial growth at one suture between the frontal and


parietal bones produces a characteristic flattening of the
forehead on the fused side, and a bulging of the forehead on the
non-fused side.
• Thi bbulging,
This l i or bbossing,
i iis a resultlt off compensatory
t growth th off
the contralateral (opposite side) coronal suture. Secondary
bulging of the temporal region on the ipsilateral (same) side can
also occur. The placement of the eyes within the orbit is also
typically affected. The superior margin of the orbit on the
synostosed side is raised, a feature known as Harlequin eye
d f it
deformity.
©™
H. Richard Winn, M.D. Youman’s Neurological Surgery 5th Edition. Philadelphia, PA: Elsevier 2004. p. 3300-01.

Classification: Neuroradiology, Infantile


Pathology, Unicoronal Synostosis
Q? Neuroradiology
• This x-ray demonstrates which of the following?
A. McRae’s line
B. McGregor’s line
C. Rule of Spence
D. Chamberlain’s line

B
A

©™
Used with permission from Handbook of Neurosurgery and
Neurology in Pediatrics; By Chaim B. Colen, MD, PhD.
27
A
A. Neuroradiology BOARD FAVORITE!

• The correct answer is C, rule of Spence.


• McRae’s line - foramen magnum diameter: 35 mm+/- 4
• McGregor’s line - line from the hard palate to the caudal most part of the opisthion
(odontoid should be < 4 mm above this line > 4 mm suggests basilar impression).
• Rule of Spence - if the sum of A + B ≥7mm, consider disruption of the transverse
ligament (requires rigid immobilization).
• Chamberlain’ss line - diagonal line from the hard palate to the posterior foramen
Chamberlain
magnum (odontoid should not extend 1/3 of its height above this line).

©™
Colen CB, Handbook of Neurosurgery and Neurology in Pediatrics, 2006.

Classification: Neuroradiology, Rule of


Spence, Clinical Definition
Q? Neuroradiology
• What is the most likely
diagnosis depicted in this MRI
and MRS?
A. Low grade glioma
B. Pleiomorphic xantho-
astrocytoma
C. High grade glioma
D None
D. N off the
h above
b

©™

34
A
A. Neuroradiology BOARD FAVORITE!

• The correct answer is C, high grade glioma.


• N-acetylaspartate (NAA) is predominantly located in neurons and is thus decreased in all
neoplasms
l th
thatt cause th
the neurons tto bbe di
displaced
l d or replaced
l d withith malignant
li t cells.
ll FiFindings
di
of numerous studies have demonstrated decreased NAA values in glial neoplasms.
• Choline (Cho) peak contains contributions from glycerophosphocholine, phosphocholine, and
phosphatidylcholine components that are thought to reflect cellular membrane density and
turnover. As in any process that leads to hypercellularity and increased membrane
proliferation the Cho value is consistently elevated in gliomas.
proliferation, gliomas
• Lactate (Lac) indicates that cellular respiration has shifted from the oxidative metabolism of
carbohydrates to nonoxidative metabolism. Increased reliance on anaerobic glycolysis is
found in highly malignant tumors.

©™
Law M, Hamburger M, Johnson G, Inglese M, Londono A, Golfinos J, Zagzag D, Knopp EA. Differentiating surgical from non-surgical lesions using
perfusion MR imaging and proton MR spectroscopic imaging. Technol Cancer Res Treat. 2004 Dec;3(6):557-65. Review.

Classification: Neuroradiology, Magnetic


Resonance Spectroscopy
Spectroscopy, High Grade
Glioma
Q? Neuroradiology
• A 64 year-old black male presented with left sided headache and slurring of speech.
Head CT is shown. Which of the following would likely be found on physical
examination?
i ti ?
A. Contralateral hemiparesis
B. Ipsilateral hemiparesis
C. Restricted left (lateral) gaze
D Restricted right (medial) gaze
D.
E. Locked-in-syndrome
F. Both A and C

©™

49
A
A. Neuroradiology
• The correct answers are A, (C and F).
• This patient has Millard Gubler syndrome!
• REMEMBER - 7th nerve (7 letters in Millard) and - 6th nerve (6 letters in Gubler)
• PLUS the corticospinal tract.
• Millard-Gubler syndrome is associated with abducens (CN6) and facial nerve
(CN7) paralysis, as well as contralateral hemiplegia of the extremities. It involves
unilateral damage to the inferior pons,
pons commonly caused by pontine infarction or
hemorrhage, and leads to damage of the above structures. The muscles of the
ipsilateral side of the face are paralyzed, diplopia, internal strabismus, and loss of
extroversion are also typically present.

©™
Onbas O, Kantarci M, Alper F, Karaca L, Okur A. Millard-Gubler syndrome: MR findings. Neuroradiology. 2005 Jan;47(1):35-7.

Classification: Neuroradiology, Millard


Gubler Syndrome, Physical Exam
Q? Neuroradiology
• A 57 year-old male underwent transnasal/
transfacial ethmoidectomy for metastatic
squamous cellll carcinoma.
i P
Post-operatively
t ti l hhe
developed clear rhinorrhea. Lumbar pucture was
performed daily for 3 days draining 20 cc each
time. On the third day he became confused and
lethargic. On examination he was non-verbal and
l li d tto painful
localized i f l stimulus.
ti l H Headd CT iis shown.
h
What is his most likely diagnosis?
A. Recurrent tumor
B. Meningitis
C. Tension pneumocephalus
D. Normal postoperative scan ©™

50
A
A. Neuroradiology
• The correct answer is C, tension pneumocephalus.
• This head CT shows the Mt Fuji sign - twin peaks with bifrontal pneumocephalus.
The Mount Fuji sign is a finding that can be observed on computed tomographic (CT)
scans of the brain (1), in which bilateral subdural hypoattenuating collections cause
compression and separation of the frontal lobes. The collapsed frontal lobes and the
widening of the interhemispheric space between the tips of the frontal lobes have the
appearance of the silhouette of Mount Fuji hence, the Mount Fuji sign.
• Given this patient’s decline in mental status he is likely developing tension
pneumocephalus.
pneumocephalus

H. Richard Winn, M.D. Youman’s Neurological Surgery 5th Edition. Philadelphia, PA: Elsevier 2004. p. 616.
©™
Ommaya AK: Cerebrospinal fluid fistula and pneumocephalus. In: Wilkins RH, Rengachary SS, eds. Neurosurgery. 2nd ed. New York: McGraw-Hill; 1996: 2773-82

Classification: Neuroradiology, Tension


Pneumocephalus, CT scan
Q? Neuroradiology
• A 4 year-old boy is brought to the
emergency room after a fall. Lateral
skull
k ll x-ray is
i shown
h tto th
the right.
i ht
Which of the following is the most
likely diagnosis?
A. Epidermoid
p
B. Eosinophilic ggranuloma
C. Hemangioblastoma
D. Normal skull x-ray

©™

66
A
A. Neuroradiology BOARD FAVORITE!

• The correct answer is B, eosinophilic granuloma.


• AP view of the skull reveals an approximate 3 cm. circular lytic lesion overlying the left
f t
frontoparietal
i t l region
i withith wellll ddelineated
li t d non-scleroticl ti margins,
i suggestive
ti off hi
histiocytosis
ti t i X
(eosinophilic granuloma).
• Differential Diagnosis: Solitary lytic lesion in the skull includes metastatic lesion, myeloma,
fibrous dysplasia, tuberculosis, trauma, histiocytosis X, osteomyelitis, leptomeningeal cyst,
and epidermoid/dermoid.
• E i
Eosinophilic
hili granuloma
l - lytic
l ti llesion
i withith wellll ddelineated
li t d non-sclerotic
l ti margins i andd
beveled edges.
• Epidermoids - lytic lesion with sclerotic margins.

©™
Yang JT, Chang CN, Lui TN, Ho YS. Eosinophilic granuloma of the skull--report of four cases. Changgeng Yi Xue Za Zhi. Dec 1993;16(4):257-62.

Classification: Neuroradiology, Pediatric


Pathology, Eosinophilic Granuloma
Q? Neuroradiology
• Treatment for this lesion in a patient with
neurofibromatosis type -1 should consist of:
A. Removing the optic nerve and
attached globe.
B. Removing the optic nerve and both
globes.
C. Radiotherapy with 25 Gray
D. Radiotherapy with 80 Gray

©™

67
A
A. Neuroradiology BOARD FAVORITE!

• The correct answer is A, removing the optic


nerve and attached globe.
• MRI demonstrates a high intensity lesion of
the distal right optic nerve, suggestive of
optic glioma, especially in a patient with
neurofibromatosis type 1.
• p gglioma distal to the
Treatment of optic
chiasm involves removing the optic nerve
and attached globe.
• Treatment of optic glioma involving the
chiasm involves removing the optic nerve,
globe and radiotherapy (less than 8 Gray).
Gray)
Board Favorite! ©™
Hollander MD, FitzPatrick M, O'Connor SG, et al: Optic gliomas. Radiol Clin North Am 1999 Jan; 37(1): 59-71.

Classification: Neuroradiology, NF-1 MRI,


Treatment Modalities
Q? Neuroradiology
• This tumor most likely has elevated serum
levels of which of following:
A. Alpha-fetoprotein
B. Carcinoembryonic antigen (CEA)
C. Cancer antigen-125 (CA-125)
D. β-human chorionic gonadotropin (β-
HCG)
E. Placental alkaline phosphatase (PLAP)

©™

69
A
A. Neuroradiology BOARD FAVORITE!

• The correct answer is D, β-HCG. Teilum Concept


• T1W non-contrast MRI sagittal image shows a pineal
recess mass compressing the tectal plate and
causing triventricular hydrocephalus.
• INTRACRANIAL GERM CELL TUMORS
• Germinoma is the most common.
• Most common tumor of pineal recess 40%
• Males
• Respond well to chemo and radiotherapy
• Cancer antigen-125 (CA-125)- ovarian cancer
• Placental alkaline phosphatase (PLAP) - ovarian
cancer

©™
Colen CB, Handbook of Neurosurgery and Neurology in Pediatrics, 2006 p. 41

Classification: Neuroradiology, Intracranial


Germ Tumors, Serum levels
Q? Neuroradiology
• A 20 year-old female presents
with 3 weeks of headaches,
nausea, andd vomiting.
iti MRI iis
shown here. Which of the
following is the most appropriate
next step in management?
A. Operative resection
B. No intervention needed
C. Shunt placement or third
ventriculostomy
D. Radiotherapy
E A andd C
E.
©™

70
A
A. Neuroradiology BOARD FAVORITE!

• The correct answer is….


• To obtain the answer to this question and to view over 250 more comprehensive
neuroradiology questions please purchase the full product here !

©™
Barkovich AJ, Krischer J, Kun LE, et al: Brain stem gliomas: a classification system based on magnetic resonance imaging. Pediatr Neurosurg 1990-91
;16(2): 73-83

Classification: Neuroradiology, Tectal


Glioma, Treatment Modalities
Q? Neurology
• This patient most likely sustained
damage to which of the following
structures?
t t ?
A. Bilateral hypoglossal nuclei
B. Left hypoglossal nerve
C. Left hypoglossal nucleus
D.
D Left upper motor neuron to the
hypoglossal nucleus
E. Right vagal nucleus

©™

2
A
A. Neurology BOARD FAVORITE!

• The correct answer is D, left upper motor neuron to the hypoglossal nucleus.
• Damage to the left hypoglossal nucleus (or nerve) would cause tongue
deviation to the left (ipsilateral). However, as depicted in the photo, damage
to the upper motor neurons (which cross), would cause tongue deviation to
the right (contralateral).

©™
Midstokke S, Hess SJ, Saini T, Edwards PC. Unilateral tongue atrophy. Gen Dent. 2006 Nov-Dec;54(6):425-7.

Classification: Neurology, upper motor


neuron lesion, hypoglossal nucleus
Q? Neurology
• The function of the superior olive is:
A. Sound localization
B. Proprioception
C. Vestibular localization and function
D. Visual gaze control

©™

6
A
A. Neurology BOARD FAVORITE!

• The correct answer A, sound localization.


• The superior olivary nucleus (or superior
olive)
li ) is
i a smallll mass off gray substance
b t Superior and Inferior colliculi

situated on the dorsal surface of the lateral


part of the trapezoid body.
• The superior olivary nucleus’ primary input
Superior Olivary nucleus
is bilateral and from the bushy cells of the
anterior ventral cochlear nuclei (AVCN)
(AVCN). Cochlear nucleus

This input occurs primarily via the ventral Nucleus gracilis


acoustic stria. Its output is to the lateral
Nucleus cuneatus
lemnisci.
• The superior olivary nucleus is the first point
where binaural input is combined.
©™
Kandel ER, Schwartz JH, Jessell TM. Principles of Neural Science, 4th ed. McGraw-Hill, New York. 2000. p. 606.

Classification: Neurology, Superior Olive


Nucleus, Function
Q? Neurology
• The function of the superior olive is demonstrated by
which waveform:
A. 1
B. 2
45
C. 3 7
D. 4
23 6
1
E 5
E.
F. 6
G. 7

©™

7
A
A. Neurology BOARD FAVORITE!

• The correct answer C, 3.


• Brainstem auditory evoked responses test the
i t it off the
integrity th auditory
dit pathway.
th
• Wave 1 –auditory nerve 45
• Wave 2 –Cochlear nuclei (pons) 7
• Wave 3 –Superior olivary complex 23 6
• Wave 4 –Lateral lemniscus 1
• Wave 5 –Inferior colliculus
• Wave 6 –Medial geniculate nucleus
• Wave 7 –Auditory radiations (cortex)
• Damage to one of the structures will result in KNOW THESE WAVEFORMS WELL!
BOARD FAVORITE!
an increased latency! (e
(e.g.
g acoustic neuroma
will show increased latency of 1-3). ©™

Kandel ER, Schwartz JH, Jessell TM. Principles of Neural Science, 4th ed. McGraw-Hill, New York. 2000. p. 606.

Classification: Neurology, Evoked


Potential, Superior Olive
Q? Neurology
• Match the following statements with the correct answer:
1. Lateral vestibulospinal tract
2. Rubrospinal tract

A. Extensor tone
B. Flexor tone
C. Both
D. Neither

©™

9
A
A. Neurology BOARD FAVORITE!

1. The correct answer is A, extensor tone. The lateral vestibulospinal tract regulates
extensor tone through the lateral vestibular nucleus.
• The vestibulospinal tract arises from the lateral vestibular nucleus (i.e. Deiter’s
nucleus) and descends bilaterally in the anterior part of the lateral funiculus.
2. The correct answer is B, flexor tone. Flexor activity is regulated by the rubrospinal
tract from the red nucleus.
• The rubrospinal tract arises from magnocellular neurons in the red nucleus and
crosses at the ventral tegmental decussation. Stimulation of the red nucleus
leads to excitation of contralateral flexor alpha motor neurons and inhibition of
extensor alpha motor neurons.

©™
Kandel ER, Schwartz JH, Jessell TM. Principles of Neural Science, 4th ed. McGraw-Hill, New York. 2000. p. 668.

Classification: Neurology, Motor Tracts,


lateral vestibulospinal and rubrospinal tract
Q? Neurology
• Which brainstem nuclei release serotonin?
A. Raphé nuclei
B. Vestibular nuclei
C. Hypoglossal nuclei
D. Nucleus cuneatus
E. Nucleus gracilis

©™

11
A
A. Neurology
• The correct answer is A, raphé nuclei. The raphé nuclei are located in the brainstem.

Neurons in the dorsal raphé


nucleus produce serotonin,
they have long projections
(green arrow) that carry the
neurotransmitter to the orbital
prefrontal cortex.

CROSS SECTION

Dorsal raphé nuclei

©™

Dorsal raphé nuclei

Classification: Neurology, Brainstem


Nuclei, Serotonin Releasing Cells
Q? Neurology
• Lack of sensory loss, presence of a “pinch sign”, and hand weakness is characteristic
of:
A. Carpal tunnel syndrome
B. Anterior interosseous syndrome
C. Ulnar nerve entrapment
D. Posterior interosseous syndrome

©™

32
A
A. Neurology BOARD FAVORITE!

• The correct answer is B, anterior interosseous syndrome. The “pinch sign” often occurs when there
is a complete anterior interosseous nerve (AIN) lesion. Attempts to pinch the tips of the terminal
phalanges of the index finger and thumb results in an extension of the distal phalanges.
phalanges Thus
Thus, the
pulps rather than the tips of these two digits approximate.

NORMAL PINCH SIGN
Note the distinctive findings
g of a AIN syndrome
y on pperformingg the ppinch test. There is loss of function in the AIN-
innervated flexor pollicis longus (FPL) and flexor digitorum profundus (FDP) muscles, leading to weakness in the
distal phalanges of the thumb and forefinger. The left photograph shows the normal function when attempting this hand ©™
posture.
H. Richard Winn, M.D. Youman’s Neurological Surgery 5th Edition. Philadelphia, PA: Elsevier 2004. p. 3925..

Classification: Neurology, Diagnostic Tests,


anterior interosseous syndrome
Q? Neurology
• 30 year-old female presents with a history of shunted pseudotumor cerebri and small
ventricles, now complains of worsening headache. Fundoscopic exam is shown below.
Whi h off the
Which th following
f ll i statements
t t t iis TRUE?
A. It is best treated with Diamox.
B. She likely has shunt failure and stiff ventricles.
C. She has venous thrombosis.
D There is no cause for concern about this headache.
D. headache

©™

63
A
A. Neurology
• The correct answer is B, she likely has shunt failure and stiff ventricles. A low index of
suspicion should be present when evaluating a patient with shunted pseudotumor cerebri
(PTC) Approximately 40% of shunts placed in children will fail in the 1st year and almost all
(PTC).
children will require shunt revision at some point. In PTC, shunt malfunction may result in
blindness.
• PTC is encountered most frequently in young, overweight women between the ages of 20 and
45. Headache, occurring in more than 90 percent of cases, is the most common presenting
complaint Dizziness
complaint. Dizziness, nausea
nausea, and vomiting may also be encountered,
encountered but there are typically
no alterations of consciousness or higher cognitive function.
• It is defined clinically by four criteria: (1) elevated intracranial pressure, as demonstrated by
lumbar puncture; (2) normal cerebral anatomy, as demonstrated by neuroradiographic
evaluation; (3) normal cerebrospinal fluid composition; and (4) signs and symptoms of
increased intracranial ppressure, includingg ppapilledema.
p
©™
Martin TJ, Corbett JJ: Pseudotumor cerebri, in Youmans JR(ed): Neurological Surgery, ed 4. Philadelphia: WB Saunders,1996, Vol 4, pp 2980–2997.

Classification: Neurology, pseudotumor


cerebri, shunt failure
Q? Neurology
• Based on the radiological features seen in this MRI,
choose the most appropriate statement regarding this
condition:
diti
A. They have no intervening brain tissue between
the vascular spaces on histopathology.
B. They do have intervening brain tissue between
the vascular spaces
p on histopathology.
p gy
C. There is no genetic association.
D. The HOX gene is highly associated.

©™

79
A
A. Neurology BOARD FAVORITE!

• The correct answer is A, they have no intervening brain tissue between the vascular spaces on
histopathology.
• Cavernous malformations (i.e.(i e angioma
angioma, cavernoma
cavernoma, or cavernous hemangioma) may be inherited or
sporadic and consist of variable sized sinusoids or cavernous spaces between capillaries. Unlike
arteriovascular malformations and capillary telangiectasias., they have no intervening brain tissue
between the vascular spaces and have thus been described as “blood sponges”. MRI shows well-
defined, usually rounded lesions with little or no mass effect and are without vasogenic edema (unless
hemorrhage is present). There may be small areas of new or old hemorrhages shown as a rim of
hemosiderosis around the cavernous angioma in the surrounding brain tissue.
tissue
• Genetics: more common in Hispanics.
• CCM1 (for cerebral cavernous malformation 1) -chromosome 7 at band 7q11.2-q21. It is also known as
KRIT1, for the protein created by the gene. 40% of familial cavernous angiomas.
• CCM2 -band 7p15-p13, protein named malcavernin. 20% of familial cavernous angiomas.
• CCM3 identified as linked to familial cavernous angioma is on chromosome 3 at band 3q. 3q
©™
Craig HD, Gunel M, Cepeda O, et al: Multilocus linkage identifies two new loci for a mendelian form of stroke, cerebral cavernous malformation, at 7p15-13 and
3q25.2-27. Hum Mol Genet 1998 Nov; 7(12): 1851-8.

Classification: Neurology, Cavernous


Malformations, Histopathology
Q
Q. Neurology
• This fundus is MOST likely seen in which of the
following patients?
A. 25 year-old female
B. 14 year-old diabetic male
C. 75 year-old diabetic male
D. 25 year-old obese male
E 25 year-old
E. ld obese
b ffemale
l

©™

80
A
A. Neurology
• The correct answer is E, 25 year-old obese female.
• This ocular fundoscopic image demonstrates classic
grade 2 papilledema most likely seen in a 25 year-
old obese female with pseudotumor cerebri
(idiopathic intracranial hypertension).

Grade II papilledema. The halo of


edema now surrounds the optic
disc.

©™
Brazis PW, Lee AG: Elevated intracranial pressure and pseudotumor cerebri. Curr Opin Ophthalmol 1998 Dec; 9(6): 27-32.

Classification: Neurology, Pseudotumor


Cerebri, Papilledema
Q? Neurology
• In myopathic disorders the motor unit potentials on electromyography would most
likely show which of the following?
A. Increased amplitude
B. Decreased duration
C. Monophasic
D. Few in numbers with decreased recruitment
E A andd D
E.

©™

86
A
A. Neurology BOARD FAVORITE!

• The correct answer is….


• To obtain the answer to this question and to view over 300 more comprehensive
neurology questions please purchase the full app here !

Polyphasic “giant wave” of reinnervation

Low amplitude wave of myopathy

©™
Daube JR: The description of motor unit potentials in electromyography. Neurology 1978 Jul; 28(7): 623-5.

Classification: Neurology,
Electromyography, Myopathic Disorders
Q
Q. Neurocritical Care
• What is the hallmark indicator for disseminated intravascular coagulation (DIC)?
A. Decreased fibrin-split products
B. Increased factor X
C. Low platelets
D. Decreased d-dimer

©™

5
A
A. Neurocritical Care
• The correct answer is C, low platelets. Coagulation factors are decreased. Fibrin-split
products and d-dimers increase.
Inciting factor: Ex. crush injury, endotoxin

Systemic activation of coagulation factors

Widespread intravascular Consumption of platelets
fibrin deposition and clotting factors

Thrombosis & organ failure Thrombosis & organ failure

©™

DIC

Classification: Neurocritical Care,


Coagulation
g Disorders,, DIC
Q? Neurocritical Care
Neurocritical Care
• A 36 year old male presents to the emergency room as a trauma code. During
resuscitation, clear fluid is noted dripping out of the left nare. An appropriate statement
regarding
di theth bedside
b d id glucose
l test
t t is
i that
th t it has:
h
A. high sensitivity, low specificity
B. high sensitivity, high specificity
C. low sensitivity, low specificity
D high sensitivity,
D. sensitivity low specificity

©™

22
A
A. Neurocritical Care
Neurocritical Care
• The correct answer is C, low sensitivity, low specificity. The bedside glucose test has low
sensitivity and low specificity. Therefore, it should be used only for trying to rule out a CSF
leak rather than ruling it in.
in
• In the emergent setting, when evaluating a patient with rhinorrhea, if the rhinorrhoea
contains glucose, the specificity of the test for CSF can be improved by excluding other
factors that increase the glucose concentration of nasal discharge. If the nasal discharge is
not blood stained, the blood glucose (measured at the same time as CSF glucose) is <6
mmol L–11, and there are no other symptoms of upper respiratory tract infection
mmol.L infection, such as
sneezing, nasal blockage, cough, sore throat, sputum, or purulent nasal discharge, then this
increases the likelihood that the discharge contains CSF. If rhinorrhoea does not
contain glucose, then either it does not contain CSF or CSF glucose concentrations are
below the limit of detection by the sticks (false negative measurement).
• g y sensitive and specific
Beta-2 transferrin test is both highly p but would require
q a few days
y to
process the Western blot at most institutions. ©™
Chan DT, Poon WS, IP CP, Chiu PW, goh KY. How useful is glucose detection in diagnosing cerebrospinal fluid leak? The rational use of CT and Beta-2
transferrin assay in detection of cerebrospinal fluid fistula. Asian J Surg. 2004 Jan;27(1):39-42.

Classification: Neurocritical Care, CSF


Leak,, Bedside Glucose Test
Q? Neurocritical Care
Neurocritical Care
• Name three functions of platelets:
A. Attach, agglutinate, agranulate
B. Adhere, activate, avascularize
C. Adhere, aggregate, agranulate
D. Apoptosis, aggregate, agranulate
E. Acidification, activate, agranulate

©™

34
A
A. Neurocritical Care
Neurocritical Care
• The correct answer is C, adhere, aggregate, and agranulate (degranulate) (Hint AAA).
Vascular
Vascular phase
phase Platelet
Platelet phase
phase
Adherence
Adherence
Vessel
V
Vessell IInjury
j
Injury Platelet

GPIb-IX-V complex
GPIb receptor Endothelial cell
vWf
Vasospasm
Vasospasm

2 Collagen

Agranulation
g
Agranulation
•Platelets can be activated by binding to collagen, Release of platelet-
+ GpIIb/IIIa
which is mediated directly through glycoprotein VI activation factors
+
(GPVI) or indirectly via von Willebrand factor (VWF) Phospholipase
C activation
binding to GPIb-V–IX. Signal transduction from any
of these receptors leads to phospholipase C GPIb-IX-V complex
recruitment, which mediates calcium mobilization,
1
platelet shape change, degranulation, and activation of fibrinogen
GPIIb/IIIa to allow binding of fibrinogen and platelet- Adherence
Adh
Adherence
3 Aggregation
platelet interactions. Aggregation ©™
Yee DL, Bergeron AL, Sun CW, Dong JF, Bray PF. Platelet hyperreactivity generalizes to multiple forms of stimulation.
J Thromb Haemost. 2006 Sep;4(9):2043-50

Classification: Neurocritical Care,


Coagulation
g Cascade,, Platelet Function
Q? Neurocritical Care
Neurocritical Care
1. Choose the most common inherited cause of post-operative bleeding?
A. Bernard-Soulier disease
B. Von Willebrand disease
C. Hemophilia A
D. Hemophilia B
E. Leiden factor disease

2. Inheritance of this disease is mostly:


A. Autosomal dominant
B. Autosomal recessive

©™

36
A
A. Neurocritical Care
Neurocritical Care
1. The correct answer is B, von Willebrand disease.
2. The correct answer is A, autosomal dominant.
• Von Willebrand disease is the most common coagulation disorder. Inheritance is
autosomal dominant transmission in 90% of cases. Children born with the disease have
either low levels (quantitative defect) of von Willebrand factor (vWF), or defective form
(qualitative defect) of the same protein.
• Bernard-Soulier: defective or decreased expression of the glycoprotein Ib/IX/V complex on
th surface
the f off th
the platelets.
l t l t Thi
This complex
l iis th
the receptor
t ffor von Will
Willebrand
b d ffactor
t ((vWF),
WF) andd
the result of decreased expression is deficient binding of vWF to the platelet membrane at
sites of vascular injury, resulting in defective platelet adhesion.
• Factor V Leiden is the most common hereditary hypercoagulability disorder. In this
disorder the Leiden variant of factor V, cannot be inactivated by activated protein C.

©™
Franchini M. Advances in the diagnosis and management of von Willebrand disease. Hematology. 2006 Aug;11(4):219-25.

Classification: Neurocritical Care,


Coagulation Disorders, Von Willebrand
Disease
Di
Q? Neurocritical Care
• You are consulted on a 22 year-old female who is one
day postpartum and has been admitted to the intensive
care unitit with
ith widespread
id d petechiae,
t hi purpura, andd
mucocutaneous bleeding. Head CT is shown on the
right. Which of the following is INCORRECT regarding
this condition?
A. Blood clotting mechanisms are activated
throughout the body.
B. Thrombocytopenia may occur.
C. Thrombocytosis is common
D. Fibrin-split products are increased.

©™

38
A
A. Neurocritical Care
• The correct answer is C, thrombocytosis is common
• Thrombocytopenia occurs, not thrombocytosis.
• The immediate postpartum period is a highly thrombogenic state. There are many risk factors
(amniotic fluid) that can cause a dysregulated hypercoagulable condition resulting in disseminated
intravascular coagulation (DIC). DIC occurs when blood clotting mechanisms are activated
throughout the body. Small blood clots form throughout the body, exhausting the supply of blood
clotting factors which become rare at sites of real tissue injury. Simultaneously, clot dissolving
mechanisms are also increased. Hence,, this disorder can result in both excessive bleedingg and
clotting. DIC may be caused by infections (especially gram-negative bacteria) , severe trauma,
cancer, blood transfusions, and obstetrical complications.
• Pathophysiology: two main mechanisms- generation of thrombin (microvascular thrombosis and
organ ischemia) and generation of plasmin (characterized mainly by hemorrhagic symptoms).
• Fibrin-split products are increased in DIC.
©™
Levi M, de Jonge E, van der Poll T, ten Cate H. Advances in the understanding of the pathogenetic pathways of disseminated intravascular coagulation result in more
insight in the clinical picture and better management strategies. Semin Thromb Hemost. 2001 Dec;27(6):569-75. Review.

Classification: Neurocritical Care,


Coagulation
g Disorders,, Post Partum
Q? Neurocritical Care
Neurocritical Care
• This ocular fundus is MOST likely seen in which
of the following patients?
A. 25 year-old obese female punched in the
stomach with a GCS of 15
B. 14 year-old male status post temporal
lobectomy
C. 31 year
year-old
old male thrown from a 3rd story
window with a GCS of 5
D. 56 year-old with a 1 cm cerebellar
hematoma with a GCS of 15

©™

46
A
A. Neurocritical Care
Neurocritical Care
• The correct answer is C, 31 year-old male thrown from a
3rd story window with a GCS of 5.
• Thi type
This t off papilledema
ill d is
i seen ini the
th setting
tti off hi
highh
intracranial pressure (ICP), such as the above patient who
has suffered severe traumatic brain injury.
• Cerebellar hematoma acutely would cause high ICPs in
the infratentorial compartment that potentially could result
in acute obstructive hydrocephalus with papilledema,
papilledema but
this is less likely in a patient that has a 1 cm cerebellar
hematoma and a GCS of 15. Grade IV papilledema. There is severe swelling in addition 
to a circumferential halo. The edema covers major blood 
vessels as they leave the optic disk (grade III) and vessels on 
the disk (grade IV).
A subretinal hemorrhage is present at 7 o
A subretinal hemorrhage is present at 7 o'clock
clock. 
©™
Bhatt UK. Bilateral optic disc swelling; is a CT scan necessary? Emerg Med J. 2005 Nov;22(11):827-30.

Classification: Neurocritical Care,


Hematoma,, Ocular Changes g
Q? Neurocritical Care
Neurocritical Care
• Which of the following is the preferred ventilation pattern in non-paralyzed patients
with central nervous system dysfunction?
A. Continuous mandatory ventilation
B. Assist control ventilation
C. Intermittent mandatory ventilation
D. Synchronous intermittent mandatory ventilation
E Pressure
E. P supportt ventilation
til ti

©™

47
A
A. Neurocritical Care
Neurocritical Care
• The correct answer is D, synchronous intermittent mandatory ventilation.
• Patients with CNS dysfunction tend to exhibit variations in tidal volume and respiratory
drive.
• Synchronized intermittent mandatory ventilation (SIMV) delivers volume-cycled
breaths that coincide with spontaneous lung inflations. The patient can breathe
unassisted and spontaneously between mechanical breaths. Also, a preselected
p y rate can be used if the ppatient`s respiratory
respiratory p y drive is insufficient to pprevent
hypercarbia.

©™
Stock MC, Perel A: Handbook of mechanical ventilatory support. Baltimore: Williams & Wilkins; 1997.

Classification: Neurocritical Care, CNS


dysfunction,
y , breathingg ppatterns
Q? Neurocritical Care
Neurocritical Care
• Which of the following characteristics are seen in patients with acute respiratory
distress syndrome (ARDS)?
A. Bilateral lung infiltrates
B. Pulmonary capillary wedge pressure less than or equal to 18 mm Hg
C. PAO2/FIO2 ratio of < 200 mm Hg
D. All of the above

©™

53
A
A. Neurocritical Care
Neurocritical Care BOARD FAVORITE!

• The correct answer is D, all of the above.


• Acute respiratory distress syndrome (ARDS) is characterized by acute onset, bilateral
infiltrates in chest x-ray, pulmonary capillary wedge pressure (PCWP) less than or
equal to 18 mm Hg and a PAO2/FIO2 ratio < 200 mm Hg.

©™
Kollef MH, Schuster DP: The acute respiratory distress syndrome. N Engl J Med 1995 Jan 5; 332(1): 27-37.

Classification: Neurocritical Care,


Respiratory Distress Syndrome, Lab
Vl
Values
Q? Neurocritical Care
Neurocritical Care
• Which of the following findings will distinguish cardiogenic pulmonary edema from
acute respiratory distress syndrome (ARDS)?
A. Bilateral lung infiltrates on chest x-ray
B. Hypoxemia
C. Pulmonary capillary wedge pressure (PCWP) > 18 mm Hg
D. All of the above

©™

55
A
A. Neurocritical Care
Neurocritical Care BOARD FAVORITE!

• The correct answer is C, Pulmonary capillary wedge pressure (PCWP) > 18 mm Hg.
• Occasionally it is very difficult to distinguish between ARDS and cardiogenic
pulmonary edema. Both can present with hypoxia and bilateral lung infiltrate, but
PCWP is > 18 mm Hg in cardiogenic pulmonary edema.

Ware LB, Matthay MA: The acute respiratory distress syndrome. N Engl J Med 2000 May 4; 342(18): 1334-49.
©™

Classification: Neurocritical Care, Acute


Respiratory Disress Syndrome, Pulmonary
Changes
Ch
Q? Neurocritical Care
Neurocritical Care
• What is most common finding on ECG in patients with pulmonary emboli?
A. Large Q wave in lead III
B. Inverted T wave in lead III
C. Right axis deviation
D. Wide S complex in lead I
E. Right bundle branch block
F. Sinus tachycardia

©™

63
A
A. Neurocritical Care
Neurocritical Care BOARD FAVORITE!

• The correct answer is F, sinus tachycardia.


• All of the answers can be seen in case of pulmonary emboli. The classic findings of
“S1,Q3,T3” are not very sensitive in diagnosis of acute pulmonary embolism.

Feied CF: Pulmonary embolism. In: Rosen and Barkin, eds. Emergency Medicine Principles and Practice. Vol 3. 4th ed. 1998:chap 111.
©™

Classification: Neurocritical Care,


Pulmonaryy Emboli,, ECG Changes g
Q? Neurocritical Care
Neurocritical Care
• A 21 year-old patient with history of recurrent sinusitis developed an allergic reaction
after transfusion of a cross matched blood. What is the most likely cause?
A. Selective IgA deficiency
B. Lab error
C. Rh group incompatibility
D. Rapid transfusion of blood products

©™

66
A
A. Neurocritical Care
Neurocritical Care BOARD FAVORITE!

• The correct answer is A, selective IgA deficiency.


• Selective IgA deficiency syndrome is the most common primary immunodeficiency.
Usually the patient is asymptomatic, but a history of recurrent infections may be
present; most commonly ear infections, sinusitis, and pneumonia. Allergies are
another possible symptom including allergic reaction to blood products.
• If only fever develops within 1-6 hours of receiving cross matched blood, this is
caused byy antibodies in recipient
p blood that react to donor leukocytes.
y Leukocyte
y ppoor
red cells can be used to prevent this reaction.

Ballow M. Primary immunodeficiency disorders: antibody deficiency. J Allergy Clin Immunol. Apr 2002;109(4):581-91.
©™
Zhao SM. Clinical assessment of preventing febrile nonhemolytic transfusion reaction by leukocyte-depleted blood transfusion. 2002 Dec;10(6):568-70.

Classification: Neurocritical Care,


Congenital Immune Disorders, IgA Def.
S d
Syndrome
Q? Neurocritical Care
Neurocritical Care
• Which of the following is the appropriate immediate treatment in case of an intra-
operative venous air embolism?
A. Irrigation of the surgical field
B. Lower the patient’s head with lateral left decubitus position
C. Aspiration of air from a multiforce central venous pressure catheter
D. Manual occlusion of the jugular veins
E All off the
E. th above
b

©™

70
A
A. Neurocritical Care
Neurocritical Care BOARD FAVORITE!

• The correct answer is E, all of the above.


• Intraoperative venous air embolism (VAE) has a high incidence during procedures
performed in the sitting position. VAE is characterized by the development of
bronchoconstriction, hypoxia, hypercarbia, hypotension, shock, cardiac arrhythmias,
increased airway pressure, and decreased end tidal CO2.
• The most sensitive diagnostic modality for VAE is transesophageal
echocardiography.
g p y

©™
Ballki M.Venous air embolism during awake craniotomy in a supine patient.Can J Anaesth. 2003 Oct;50(8):835-8.

Classification: Neurocritical Care,


Intraoperative Venous Embolism,
T t
Treatment t
Q? Neurocritical Care
Neurocritical Care
• Match the following arterial blood gas values with the appropriate acid/base disorder.
A. Metabolic acidosis
B. Metabolic alkalosis
C. Respiratory acidosis
D. Respiratory alkalosis
E. Combined respiratory acidosis and metabolic acidosis
1. pH 7.26, PaCO2 53 mmHg, HCO3- 24 mEq/L
2. pH 7.49, PaCO2 46 mmHg, HCO3- 33 mEq/L
3. pH 7.30, PaCO2 33 mmHg, HCO3- 18 mEq/L
4. pH 7.27, PaCO2 41 mmHg, HCO3- 18 mEq/L

©™

75
A
A. Neurocritical Care
Neurocritical Care BOARD FAVORITE!

• The correct answers are….


• To obtain the answer to this question and to view over 250 more comprehensive
neurocritical care questions please purchase the full product here !

Narins RG, Emmett M: Simple and mixed acid-base disorders: a practical approach. Medicine (Baltimore) 1980; 59(3): 161-87.
©™

Classification: Neurocritical Care, Acid


Base Disorders,, Lab Values
Neuropharmacology Section

©™
Q? Neuropharmacology
• A young man is brought to your clinic who has a history of seizures. He stopped taking
his medication for a while, but then decided to “double up” after having a seizure
recurrence a ffew monthsth ago. UUpon physical
h i l exam, you noticeti ataxia,
t i slurred
l d speech,
h
and gingival proliferation.
• Which of the following seizure drugs is associated with the side effects mentioned
above?
A. Carbamazepine p
B. Phenytoin
C. Valproate
D. Zolpidem
E. Phenobarbital
©™

1
A
A. Neuropharmacology
• The correct answer is B, Phenytoin.
• Phenytoin acts by stabilizing the inactive state of calcium channels.
• Although the other agents mentioned cause various cognitive shifts, only
phenytoin causes gingival proliferation.
• Carbamazepine toxicity may cause hyponatremia, ataxia, nystagmus,
slurring of speech, dystonia, and varying degrees of CNS depression.
• Valproate toxicity may cause coma,
coma confusion
confusion, somnolence
somnolence, and cerebral
edema.
• Zolpidem toxicity may cause psychosis, distortion in visual perception, and lethargy.
• Phenobarbital toxicity can cause fatal respiratory depression, nystagmus, and ataxia.

©™
Amstrong L, Goldman M, Lacy C, Lance L. Drug Information Handbook. 13th Edition. 2005. pp 1195-1199

Classification: Neuropharmacology,
antiseizure medication,, pphenytoin
y toxicityy
Q? Neuropharmacology
• A 61 year-old woman presents to your office complaining of shooting pains on the left
side of her face. The attacks of pain are so severe that she avoids brushing her teeth
on the
th left
l ft side,
id andd describes
d ib multiple
lti l attacks
tt k throughout
th h t the
th day.
d
• Which of the following drugs would be considered first line treatment for her condition?
A. Oxycodone
B. Phenytoin
C Ethosuximide
C.
D. Carbamazepine
E. Phenobarbital

©™

Classification: Neuropharmacology,
antiseizure medication,, pphenytoin
y toxicityy
A
A. Neuropharmacology
• The correct answer is D, Carbamazepine.
• The patient is suffering from trigeminal neuralgia, which is best treated with
carbamazepine.
• Carbamazepine is related to the structure of tricyclic antidepressants.
• The method of action of this drug is related to the inhibition of voltage sensitive
calcium channels and the stabilization of sodium channels.
• Trigeminal neuralgia is a debilitating disease characterized by neuropathy of the fifth
cranial nerve (trigeminal nerve).
• Intense pain is experienced in the jaw, scalp, forehead, eyes, and nose.
• Most patients who develop trigeminal neuralgia are over 40 years-old.
• None of the other drugsg listed are considered first line therapy
py for this condition.
©™

A t
Amstrong L,
L GGoldman
ld M
M, LLacy C
C, LLance LL. D
Drug IInformation
f ti H Handbook.
db k 13th Edition.
Editi 2005.
2005 pp 255
255-257
257

Classification: Neuropharmacology,
trigeminal
g neuralgia,
g , carbamazepine
p
Q? Neuropharmacology
• An eight year old boy is brought to your clinic. His mother mentions that her son often
drifts off in space, and has memory lapses during this period. EEG reveals a 3/second
spike
ik andd wave pattern.
tt
• You prescribe ethosuximide for this patient. Which of the following is the most likely
mechanism of action for this drug?
A. Reduces current in T-type calcium channels on primary afferent neurons
B. Activates a hyperpolarizing potassium current and reduces voltage gated sodium
currents
C. Inhibits neuronal and glial uptake of GABA
D. Enhances the inhibitory action of the GABA receptor.

©™

3
A
A. Neuropharmacology
• The correct answer is A, reduces current in T-type calcium channels on primary
afferent neurons.
• This patient is presenting with classic absence seizures.
• Ethosuximide is a first line treatment for absence seizures. It reduces current in T-
type calcium channels on primary afferent neurons. Adverse effects include lethargy
and GI problems.
• Topiramate activates a hyperpolarizing potassium current
current, reduces voltage gated
sodium current, and increases postsynaptic GABA-A receptor currents.
• Tiagabine inhibits neuronal and glial uptake of GABA. It is used for treatment of
partial seizures.
• Phenobarbital increases the inhibition of the GABA channel by increasing the length
off titime th
the chloride
hl id channel
h l remains
i open.
©™
Amstrong L, Goldman M, Lacy C, Lance L. Drug Information Handbook. 13th Edition. 2005. pp 1488-1489

Classification: Neuropharmacology,
antiseizure medication,, ethosuximide MOA
Q? Neuropharmacology
• A young mother brings her one year old baby to the emergency room. The baby has
an elevated temperature of 102°F, has been vomiting, and just had a seizure. The
b b has
baby h receivedi d no immunizations.
i i ti A spinal
i l tap
t isi performed.
f d The
Th CSF is
i cloudy
l d
and the following lab values are obtained:
CSF glucose: 30
CSF protein: 80
Opening pressure: 220
• Microbiology reports gram negative coccobacilli that are beta lactamase positive.
Which is the most appropriate antibiotic regimen to begin in this patient?
A. Vancomycin
B. Moxifloxacin
C Ceftriaxone
C.
©™
D. Penicillin G
4
A
A. Neuropharmacology
• The correct answer is C, Ceftriaxone.
• This child has Haemophilus B (HiB) meningitis.
• Although infection with HiB is declining due to the widespread use of the
vaccine, children who have not received the vaccine are still susceptible.
• Ceftriaxone is a third generation cephalosporin with coverage for HiB.
• Vancomycin is used against gram positive organisms. Haemophilus is a
gram negativeti organism.
i
• Quinolones could be used to treat HiB, but they are not recommended for use in
children due inhibition of growth at the epiphyseal plate in some studies.
• Penicillin G would not be used as lab reports indicated the bacteria, beta lactamase
resistant,, was ppresent. In addition,, ppenicillin G does not have HiB coverage.
g
©™
Amstrong L, Goldman M, Lacy C, Lance L. Drug Information Handbook. 13th Edition. 2005. pp288-289

Classification: Neuropharmacology,
antimicrobials, third generation
cephalosporins
h l i
Q? Neuropharmacology
• An eight year-old boy is taken by his father to the emergency room. An aseptic
meningitis is diagnosed by the ER physician. Physical exam shows
sores in
i th
the mouthth located
l t d on the
th tongue
t andd gums andd a skin
ki rashh presentt on the
th
palm of the patient’s hands and the soles of his feet. What treatment is most
appropriate?
A. Amantadine
B. Acyclovir
y
C. Tenofovir
D. No treatment needed

©™

5
A
A. Neuropharmacology
• The correct answer is D, no treatment needed.
• This patient has hand-foot-mouth disease, often caused by coxsackie virus A16,
which is a subgroup of the enteroviruses.
• Currently there is no specific treatment available for any of the enteroviruses.
• The clinical course of coxsackie A16 is relatively mild.
• Amantadine is indicated for treatment of type A influenza. It is also used in
Parkinson’ss disease.
Parkinson disease
• Acyclovir is the agent of choice in herpes simplex and herpes zoster infections.
• Tenofovir is a nucleotide analogue reverse inhibitor used in HIV.

©™
Amstrong L, Goldman M, Lacy C, Lance L. Drug Information Handbook. 13th Edition. 2005. pp 43, 79-80, 1435

Classification: Neuropharmacology,
Antiviral medications,, Coxsackie A16
Q? Neuropharmacology
• A 35 year-old man has suffered from severe asthma attacks for most of his life. Which
of the following agents could cause a life threatening bronchospasm for this patient?
A. Phenoxybenzamine
B. Terazosin
C. Acebutolol
D. Propanolol

©™

6
A
A. Neuropharmacology
• The correct answer is D, Propanolol.
• Propanolol is a non-selective beta blocker.
• Alpha mediated bronchoconstriction (α1) could thus take precedence over
β2 relaxation.
• Phenoxybenzamine is a non selective α-blocker primarily used for treatment of
pheochromocytoma. It causes no adverse effects in asthmatics.
• Terazosin is an α-1
α 1 adrenergic receptor antagonist for treatment of hypertension and
BPH. It is not associated with any adverse effects in asthmatics.
• Acebutolol is a β -1 antagonist with sympathomimetic activity. It is especially indicated
for patients who have asthma or diabetes.

Amstrong L, Goldman M, Lacy C, Lance L. Drug Information Handbook. 13th Edition. 2005. pp1267-1289 ©™

Classification: Neuropharmacology,
autonomic agents,
g , ppropanolol
p
Q? Neuropharmacology
• Which of the following drugs is given by intravenous infusion, has a very short half life,
and is used primarily during hypertensive crises?
A. Esmolol
B. Prazosin
C. Carvedilol
D. Yohimbine

©™

7
A
A. Neuropharmacology
• The correct answer is A, Esmolol.
• Esmolol is a β-1 receptor antagonist for use in hypertensive crises.
• It can also be used during episodes of acute supraventricular tachycardia.
• Prazosin is used for hypertension and benign prostatic hyperplasia. It is not indicated
for control of acute episodes of hypertension, however.
• Carvedilol is an alpha and beta antagonist. At this time it is used to treat heart failure.
It is not given to control a hypertensive crisis.
crisis
• Yohimbine is an a-2 agonist. At this time its clinical use is limited.

©™
Amstrong L, Goldman M, Lacy C, Lance L. Drug Information Handbook. 13th Edition. 2005. pp 546-548

Classification: Neuropharmacology,
autonomic agents,
g , Esmolol
Q? Neuropharmacology
• A 32 year-old soccer player injures his knee playing in the European World Cup. After
surgery to repair a torn ligament, he develops osteoarthritis two years later. He has a
history
hi t off allergy
ll to
t sulfa
lf drugs.
d Which
Whi h off the
th following
f ll i NSAIDS would ld nott be
b
recommended in this patient?
A. Aspirin
B. Ibuprofen
C. Sulindac
D. Piroxicam

©™

8
A
A. Neuropharmacology
• The correct answer is C, Sulindac.
• Sulindac is an NSAID that contains a sulfa moiety, and thus produces an allergic
reaction in those who are allergic to sulfa drugs.
• None of the other choices contain a sulfa moiety.

©™
Amstrong L, Goldman M, Lacy C, Lance L. Drug Information Handbook. 13th Edition. 2005. pp 1413-1414

Classification: Neuropharmacology,
Analgesics,
g , Sulindac
Q? Neuropharmacology
• Which of the following drugs is used for treatment of myasthenia gravis?
A. Edrophonium
B. Pyridostigmine
C. Echothiophate
D. Parathion
E. Donezipil

©™

10
A
A. Neuropharmacology
• The correct answer is B, Pyridostigmine.
• Pyridostigmine is a carbamate which acts as a pseudo-reversible inhibitor of
acetylcholinesterase.
• It is rapidly hydrolyzed after carbamoylating the active site of the
acetylcholinesterase enzyme.
• It is used extensively in the treatment of myasthenia gravis.
• Edrophonium is a reversible inhibitor of AchE.
AchE Used for the diagnosis of myasthenia
gravis, but not for treatment due to an extremely short half life.
• Echothiophate is an irreversible inhibitor of AchE. Primarily utilized for glaucoma.
• Parathion is an irreversible inhibitor of AChE. Primarily utilized for insect control.
• Donezipil
p is a reversible inhibitor of AChE. Primarilyy utilized for treatment of
Alzheimer’s disease. ©™
Amstrong L, Goldman M, Lacy C, Lance L. Drug Information Handbook. 13th Edition. 2005. pp 1281-1282

Classification: Neuropharmacology,
anticholinesterase drugs,
g , py
pyridostigmine
g
Q? Neuropharmacology
• Which variables alter the mechanism of intrathecal drug distribution?
A. Age
B. Weight
C. Vicosity
D. Patient position
E. All of the above.

©™

20
A
A. Neuropharmacology
• The correct answer is E, all of the above.
• Factors that influence the distribution of intrathecally administered medication
include:
1. Characteristics of the injected solution: baricity, volume/dose/concentration,
temperature of injectate, viscosity, additives
2. Clinical technique: patient position, level of injection, needle type/alignment,
intrathecal catheters,
catheters fluid currents
currents, epidural injection
3. Patient characteristics: age, height, weight, sex, intra-abdominal pressure, spinal
anatomy, lumbosacral CSF volume, pregnancy

©™
Greene NM. Distribution of local anesthetic solutions within the subarachnoid space. Anesth Analg 1985; 64: 715–30.

Classification: Neurosurgery, Intrathecal


Drugs, Variables Affecting Mechanism of
Distribution
Q? Neuropharmacology
• A 76 year-old male presents to your clinic in consultation with results of a recent
angiogram that suggest carotid stenosis 45% by NASCET criteria. He denies current
visual
i l changes,
h hheadaches
d h or weakness,
k bbutt says th
thatt 2 weeks
k ago hhe did have
h “loss
“l
of sight” in the left eye for one day . You would recommend:
A. Medical therapy (ie, antiplatelet agents and cardiovascular risk factor
control/prevention).
B. Recommend carotid endarterectomyy after consider ppatient's risk factor pprofile
and comorbidities.
C. Recommend endarterectomy without considering patient's risk factor profile and
comorbidities.
D. Carotid endarterectomy plus antiplatelet therapy and cardiovascular risk factor
control/prevention
control/prevention.
©™

21
A
A. Neuropharmacology BOARD FAVORITE!

• The correct answer is A, medical therapy.


• NASCET criteria for carotid stenosis:
• Symptomatic (TIA or minor stroke)
• Lower-grade carotid stenosis (<50% by NASCET criteria) -medical therapy (ie, antiplatelet
agents and cardiovascular risk factor control/prevention)
• Moderate-grade symptomatic carotid stenosis (50% to 69% by NASCET criteria) walk a finer
line. Consider patient's risk factor profile and comorbidities.
• High-grade symptomatic carotid stenosis (70% to 99% by NASCET criteria) -CEA plus
antiplatelet therapy and cardiovascular risk factor control/prevention is the treatment of choice.
• Asymptomatic
• >60% carotid stenosis (NASCET criteria) walk a fine line. If surgical complications exceed 3%,
CEA would be harmful to the ppatient.
©™
Ferguson GG, Eliasziw M, Barr HWK, Clagett GP, et al. The North American Symptomatic Carotid Endarterectomy Trial. Stroke. 1999;30:1751-8.

Classification: Neurosurgery,
Neurosurgery NASCET,
NASCET
Carotid Stenosis
Q? Neuropharmacology
• A 75-year old man who had just suffered
an acute myocardial infarction one week
previously now presents to clinic with
tachycardia. His EKG is shown here. An
appropriate treatment regimen would
include which of the following drugs?
A. Magnesium
ag es u
B. Procainamide
C. Amiodarone
D. Verapamil
E. Lidocaine
©™

31
A
A. Neuropharmacology
• The correct answer is….
• To obtain the answer to this question and to view over 250 more comprehensive
neuropharmacology questions please purchase the full product here !

Stoelting R, Hillier s. Pharmacology and Physiology in Anesthetic Practice 4th Edition. Philadelphia, PA: Lipincott 2006. p. 620 ©™

Category: Pharmacology, torsades de pointe, magnesium 
Q? Neurobiology
• Match the following statements
with the correct answer:
1. Neuromuscular junction A. A
A Acetylcholine
t l h li release
l
2. Botox B. Acetylcholine receptor
3. Renshaw cell C. 5 subunits: 2α subunits, a β, a δ and either
4. Blocks Renshaw cell a γ or an ε.
neurotransmitter D Strychnine
D.
5. Curare E. Glycine

©™

4
A
A. Neurobiology BOARD FAVORITE!

• The corrects answers are the following: 1 - C, 2 - A, 3 - E, 4 - D, 5 - B.


• Neuromuscular junction- the neuromuscular junction contains the nicotinic receptor
(an acetylcholine receptor subtype (nAChR)) There are 2 types of nAChRs;
autonomic and neuromuscular. The autonomic nicotinic receptor has 5 subunits: 3α
and 2β. The neuromuscular nAChR also consists of 5 subunits:2α subunits, a β, a
δ and either a γ or an ε subunit.
• Botox –botulism toxin decreases Ach release ((at neuromuscular jjunction and
autonomic nerve terminals: both sympathetic & parasympathetic) by binding to
synaptotagmin, while curare (δ-tubocurarine) blocks the Ach receptor. Botox has
actually become useful for cosmetic purposes and dystonias.
• The Renshaw cell neurotransmitter is glycine; an aminoacid and whose channel is
blocked by strychnine.
strychnine
Dressler D, AdibSaberi F. New formulation of Botox: complete antibody-induced treatment failure in cervical dystonia. J NeurolNeurosurg Psychiatry. ©™
2007 Jan;78(1):108-9
Adams, M.E. and Olivera, B.M. Neurotoxins: overview of an emerging research technology. Trends Neurosci. 17:151-155. (1994).

Classification: Neurobiology,
Neuromuscular junction, Receptor actions
Q? Neurobiology
• Which of the following is the principle visceral nucleus of the brainstem?
A. Nucleus ambiguus
B. Raphe nucleus
C. Nucleus solitarius
D. Locus ceruleus
E. Trigeminal nucleus

©™

5
A
A. Neurobiology BOARD FAVORITE!

• The correct answer is C, nucleus solitarius.


• Nucleus solitarius receives afferent visceral information from the heart, lungs, GI
tract, as well as for taste. The solitary nucleus and tract are neural structures in the
brainstem that carry and receive visceral sensation and taste from the facial (VII),
glossopharyngeal (IX), vagus (X) cranial nerves, as well as the cranial part of the
accessory nerve (XI).
• Nucleus ambiguous
g ggives rise to the efferent motor fibers of the vagus
g nerve ((CN X))
terminating in the laryngeal and pharyngeal muscles, efferent motor fibers of the
glossopharyngeal nerve (CN IX) terminating in the stylopharyngeus and contains the
preganglionic parasympathetic neurons for cardiac regulation.

©™
Andresen MC, Doyle MW, Bailey TW, Jin YH. Differentiation of autonomic reflex control begins with cellular mechanisms at the first synapse within the
nucleus tractus solitarius. Braz J Med Biol Res. 2004 Apr;37(4):549-58.

Classification: Neurobiology, Brainstem,


Nucleus Solitarius
Q? Neurobiology
• Choose the MOST accurate statement regarding the physiology of an action potential:
A. Voltage-gated potassium channels (also called delayed rectifier potassium
channels) have a delayed response, such that potassium continues to flow out
of the cell and initiate depolarization.
B. Hyperpolarization is caused by K+ influx into a cell.
C. Closing of voltage-gated potassium channels is both voltage- and time-
dependent.
D. Depolarization is caused by Na+ efflux from a cell.

©™

35
A
A. Neurobiology BOARD FAVORITE!

• The correct answer is C, closing of voltage-gated potassium channels is both voltage- and time-
dependent. As potassium exits the cell, the resulting membrane repolarization initiates the closing of
voltage-gated potassium channels.
• These channels do not close immediately in
response to a change in membrane potential. Peak
Rather, voltage-gated potassium channels 40
(also called delayed rectifier potassium

ne voltage (mV)
channels) have a delayed response, such
that potassium continues to flow out of the
cell even after the membrane has fully 0
repolarized. Thus the membrane potential K efflux +

Membran
dips below the normal resting membrane
potential of the cell for a brief moment; this
dip of hyperpolarization is known as the -55 Threshold
Resting potential
undershoot.
-70
• Hyperpolarization is caused by K+ efflux
from a cell. Stimulus Na influx Hyperpolarization +

• Depolarization is caused by Na Na+ influx into a 0 1 2 3 4 5


Time (ms)
cell.
©™
Kandel ER, Schwartz JH, Jessell TM. Principles of Neural Science, 4th ed. McGraw-Hill, New York. 2000. p. 128-33.
Robbins and Cotran. Pathologic Basis of Disease, 6th Edition. W.B. Saunders Company. 1999. p.55-56.

Classification: Neurobiology,
Neurophysiology, Action Potential
Q? Neurobiology
• Neurotransmitter that is found in a postganglionic sympathetic nerve terminal:
A. GABA
B. Serotonin
C. Acetylcholine
D. Norepinephrine
E. C + D

©™

36
A
A. Neurobiology BOARD FAVORITE!

• The correct answer is E, acetylcholine (Ach) and norepinephrine (NE).


• Both Ach and NE are found in postganglionic sympathetic fibers.
• Ach- sweat, piloerectors, and vasculature, mediated by muscarinic receptors
(inhibited by pertussis toxin).
• REMEMBER: sweating of the palms of the hands is parasympathetic NOT
sympathetic! (BOARD FAVORITE!)
• NE all others
NE-
• Peripheral sympathetic pathway –TRAVELS WITH SKELETAL NERVES (sweat,
piloerectors, and vasculature) (BOARD FAVORITE!)
• Head and Neck sympathetic pathway -TRAVELS WITH BLOOD VESSELS
(BOARD FAVORITE!)
• All preganglionic fibers (both sympathetic and parasympathetic) use Ach. ©™
Kandel ER, Schwartz JH, Jessell TM. Principles of Neural Science, 4th ed. McGraw-Hill, New York. 2000. p. 1051-52.

Classification:
Cl ifi ti N Neurobiology,
bi l
Neurophysiology, Sympathetic Nerve
Terminal
Q? Neurobiology
• Which of the following statements is true regarding the intracellular second
messenger cyclic AMP.
A. It is involved with photoreception
B. It is hydrolyzed by phospholipase
C. It synergistically activates protein kinase C
D. It is increased with D1 receptor stimulation

©™

37
A
A. Neurobiology BOARD FAVORITE!

• The correct answer is D, it is increased with D1 receptor stimulation.


• Hormones that utilize this system include: epinephrine and norepinephrine, glucagon
and TSH.
TSH
• In a cAMP dependent pathway, the activated Gs alpha subunit binds to and activates
an enzyme called adenylyl cyclase which in turn catalyzes the conversion of ATP into
cyclic adenosine monophosphate (cAMP). The intracellular second messenger cAMP
activates protein kinase A (PKA).
• cAMP is decomposed to AMP by the action of the enzyme phosphodiesterase. This
cAMP phosphodiesterase - has an activity opposite to kinase,
kinase therefore it
dephosphorylates cAMP into AMP, reducing the cAMP levels.
• Deregulation of cAMP pathways (hyperactive) and an aberrant activation of cAMP-
controlled genes is linked to the growth of some cancers
• Activation of conventional protein kinase C - requires DAG, Ca2+, and phospholipid.

©™
Kandel ER, Schwartz JH, Jessell TM. Principles of Neural Science, 4th ed. McGraw-Hill, New York. 2000. p. 230-36.

Classification:
Cl ifi ti N Neurobiology,
bi l
Neurophysiology, Second Messenger
Systems
Q? Neurobiology
• Match the receptors with their action:
1. Mu1 A. Hallucinations
2. Mu2 B. Opiate analgesic euphoria
3. Kappa C. Miosis
4. Delta D. Reinforcing behavior
5. Sigma E. Respiratory depression

©™

38
A
A. Neurobiology BOARD FAVORITE!

• The correct answers are:


1. B, Mu1- Opiate analgesic euphoria
2. E, Mu2- Respiratory depression
3. C, Kappa- Miosis
4. D, Delta- Reinforcing behavior
5. A, Sigma- Hallucinations

©™
Kandel ER, Schwartz JH, Jessell TM. Principles of Neural Science, 4th ed. McGraw-Hill, New York. 2000. p. 483-6.
Robbins and Cotran. Pathologic Basis of Disease, 6th Edition. W.B. Saunders Company. 1999. p.55-56.

Classification: Neurobiology,
Pharmacology, Receptor Actions
Q? Neurobiology
• Hexamethonium blocks which receptor?
A. Muscarinic
B. Nicotinic
C. Glycine
D. Glutamate

©™

39
A
A. Neurobiology BOARD FAVORITE!

• The correct answer is B, nicotinic receptor.


• Hexamethonium is a nicotinic Ach receptor antagonist, acts by preventing
acetylcholine from binding to the cholinergic receptor.
receptor
• It has no effect on the muscarinic acetylcholine receptors, or mAChRs.

©™
Kandel ER, Schwartz JH, Jessell TM. Principles of Neural Science, 4th ed. McGraw-Hill, New York. 2000. p. 971.

Classification: Neurobiology,
Pharmacology, Receptor Actions
Q? Neurobiology
• A 37 year-old male presented with generalized seizure and expressive dysphasia.
MRI is shown. Reoccurrence of this tumor as a high-grade is LEAST likely due to
which
hi h genetic
ti abnormality?
b lit ?
A. P53 tumor suppressor
B. Retinoblastoma gene
C. Amplification of platelet
derived growth factor
D. Amplification of epidermal
growth factor receptor

©™

41
A
A. Neurobiology BOARD FAVORITE!

• The correct answer is D, amplification of epidermal growth factor receptor.


• This MRI demonstrates a low-grade glioma. Recurrent progressive low-grade glioma is
associated
i t d with
ith mutations
t ti in
i the
th ttumor suppressor gene p53
53 with
ith amplification
lifi ti off CDK4 or
loss of Rb and amplification or overexpression of PDGF.
• There are 2 subtypes of glioblastoma
– Older patients with de novo tumors
• Deletions in cell cycle related INK4a-ARF genes p16INK4A and p19/p14ARF
• EGFR amplifications
– Younger patients with progressive low grade glioma
• Mutations in the tumor suppressor gene p53 with amplification of CDK4 or loss of
Rb
• Amplification or overexpression of PDGF
©™
Berger MS, Pardos M,Textbook of Neuro-Oncology. Saunders, Philadelphia. 2005. p. 608-9.

Classification: Neurobiology, Genetics,


Tumor Growth Factors
Q? Neurobiology
• Match the following sensory receptors with their description.
1. Meissner capsule A. Rapidly adaptive, vibration, receptors
2. Merkel receptors B. Slowly adaptive, touch and pressure, receptor
3. Free nerve endings C. Slowly adaptive, heavy pressure, receptor
4. Ruffini corpuscles D. Rapidly adaptive, touch, receptors
5. Pacinian corpuscles E. Pain receptors

©™

60
A
A. Neurobiology BOARD FAVORITE!

• The correct answers are….


• To obtain the answer to this question and to view over 250 more comprehensive neurobiology
questions please purchase the full product here !

R M

Ruffini and Merkel drink TONIC SLOWLY! (slowly adapting tonic receptors)

©™
Guyton, Arthur C., John E. Hall. Textbook of medical physiology. W.B. Saunders Company; 10th edition, 2000. p. 521, 530-2

Classification: Neurobiology,
Neurophysiology, Touch Receptors
Q? Neuroanatomy
• Which of the following statements is INCORRECT regarding the anatomy of the
cavernous carotid artery?
A. The inferior hypophyseal artery is most commonly a branch of the
meningohypophyseal trunk.
B. The inferior hypophyseal artery passes medially to the posterior pituitary capsule.
C. Persistent trigeminal arteries can also originate from the posterior vertical
segment of the cavernous internal carotid artery and pass posteriorly through the
posterior wall of the cavernous sinus to join the basilar artery between the origin
of the superior and anterior inferior cerebellar arteries.
D. The tentorial artery, or the artery of Bernasconi and Cassinari, is the most
inconstant branch of the meningohypophyseal trunk.

©™

1
A
A. Neuroanatomy
• The correct answer is D. The Inferior hypophyseal artery Meningohypophyseal trunk 
meningohypophyseal trunk is the most
inconsistent branch of the
meningohypophyseal trunk. Cerebral
Peduncle
• The meningohypophyseal trunk is the most
constant artery (labeled below). Pituitary

Tentorial artery
Carotid CN3

Dorsal meningeal artery
V1
V1
• The meningohypophyseal trunk usually CN4

arises from the posterior aspect of the


central third of the posterior bend of the
artery at the level of the dorsum sella and CN5
V2
frequently gives rise to three branches: the
tentorial artery (Bernasconi and Cassinari), Middle Fossa
th ddorsall meningeal
the i l artery,
t andd the
th inferior
i f i
hypophyseal artery. ©™
Isolan G, de Oliveira E, Mattos JP. Microsurgical anatomy of the arterial
• All other answers are true statements. compartment of the cavernous sinus: analysis of 24 cavernous sinus.
ArqNeuropsiquiatr. 2005 Jun;63(2A):259-64

Comments:  Cerebral Arteries, 
meningohypophyseal, Bernasconi and 
Cassinari
Q? Neuroanatomy
• α-motor neurons are most commonly found in which Rexed lamina?
A. III
B. V
C. VI
D. IX
E. X

©™

2
A
A. Neuroanatomy BOARD FAVORITE!

• The correct answer is D, IX.


• Rexed lamina IX, is located in the ventral motor
areas off th
the gray matter.
tt D Described
ib d iin 1950
1950s bby 1
Bror Rexed as a way to name the different parts of 2
3
the spinal cord, the Rexed laminae consist of ten 4
5
layers of grey matter (I-X). It must be noted that 10 6
Rexed areas are not designated in terms of 7
9
8
location but rather by their cellular structure.
location, structure 9
9
• Layer 1 – marginal zone, fast pain (Aδ) and temperature
fibers
• Layer 2 – substantia gelatinosa,slow pain (C fibers)
• Layer 4 – nucleus propius, interneurons to thalamus
• Layer 7 – contains the zona intermedia
• L
Layer 9 – α and
d γ motorneurons
t
• Layers are all BOARD FAVORITE topics ! ©™
Anamizu Y, Seichi A, Tsuzuki N, Nakamura K. Age-related changes in histogram pattern of anterior horn cells in human cervical spinal cord.Neuropathology. 2006
Dec;26(6):533-9.

Comments:  Spine, Rexed lamina
Q? Neuroanatomy
• Match the following cortical
neuroanatomy: F
1. Sylvian fissure
E
2. Rolandic fissure G
3. Pars triangularis H
4. Pars opercularis B C D
55. P
Pars orbitalis
bit li A
6. Supramarginal gyrus
7. Angular gyrus

©™

36
A
A. Neuroanatomy BOARD FAVORITE!

• Cortical neuroanatomy: Motor cortex Sensory cortex


1. D, Sylvian fissure F
2. E, Rolandic fissure E
3. B, Pars triangularis G
4. C, Pars opercularis H
5. A, Pars orbitalis B C D
6. G, Supramarginal gyrus A
7. H, Angular gyrus
• Note: F is the precentral sulcus.

©™

H. Richard Winn, M.D. Youman’s Neurological Surgery 5th Edition. Philadelphia, PA: Elsevier 2004. p. 2533-7.

Comments:  Cortical anatomy, fissures, gyri
Q? Neuroanatomy
• Match the following boundaries of Kawase’s triangle to their anatomic structure.
A. Anteromedial boundary
B. Anterolateral boundary
C. Posterior boundary
1. Greater superficial petrosal nerve (GSPN)
2. Internal auditory canal
3. Lateral edge of the trigeminal nerve

©™

37
A
A. Neuroanatomy
• The correct answers are A - 3, B - 1,
C - 2.
1
• The anteromedial boundary is
formed by the lateral edge of the
trigeminal nerve. 4
2 3 5
• The anterolateral boundary is
formed by the greater superficial O. C 6

petrosal nerve (GSPN).


• The posterior boundary is formed by
the internal auditory canal or a line
connecting the hiatus fallopii to the
dural ostium of Meckel’s cave.
1- Anteromedial triangle 4- Lateral triangle (Parkinson's)
2- Medial triangle 5- Posterolateral triangle (Glasscock's)
3 Superior triangle
3- 6 Posteromedial triangle (Kawase
6- (Kawase's)
s)

©™
H. Richard Winn, M.D. Youman’s Neurological Surgery 5th Edition. Philadelphia, PA: Elsevier 2004. p. 921-5.

Comments:  Kawase’s triangle, boundaries, 
CN V, Greater superficial petrosal nerve
Q? Neuroanatomy
• The vertebral arteries enter within the transverse foramina of which vertebrae:
A. C4 to C1
B. C6 to C1
C. C6 to C2
D. C7 to C2
E. C8 to C2

©™

52
A
A. Neuroanatomy BOARD FAVORITE!

• The correct answer is B, C6 to C1.


Anterior arch of atlas
• The vertebral artery passes through the transverse
Vertebral arteryy
foramina of C6 through C1 before exiting the transverse Superior articular 
Superior articular
fovea
foramina of the atlas (C1).
• At this location the vertebral artery curves posterior and
superior to lie in a groove on the upper surface of the
atlas .
• At C7 the vertebral artery swings anteriorly and laterally
because it normally runs outside the rudimentary
transverse foramina of C7. When a foramina is present,
transverse foramina of the seventh cervical vertebra Cranial view of the course of the vertebral artery
almost always contains vascular and nerve branches as over C1
well as fibrous and adipose tissues.

©™
Katsuta T, Rhoton AL Jr, Matsushima T.The jugular foramen: microsurgical anatomy and operative approaches. Neurosurgery. 1997 Jul;41(1):149-201.
MS Jovanovic. A comparative study of the foramen transversarium of the sixth and seventh cervical vertebrae. Volume 12, Number 3 / September, 1990

Comments:  Extracranial Vasculature, 
Vertebral artery, Cervical spine, transverse 
foramina 
Q? Neuroanatomy
• Match the structures associated with this portion of
the lateral ventricle:
1 Right
1. Ri ht foramen
f off Monro
M A E
2. Left foramen of Monro
3. Choroid plexus
4. Thalamostriate vein
F
5. Anterior cerebral artery
B
6. Anterior septal vein
7. Fornix C G
8. Internal cerebral vein

D
©™

17
A
A. Neuroanatomy BOARD FAVORITE!

• The correct answers are….


Head of caudate
• To obtain the answer to this question
and to view over 200 more A E
comprehensive neuroanatomy questions
please purchase the full product here !
F
B

G
C

Thalamus
D

©™

Comments:  Lateral ventricle anatomy, 
cerebral vascular anatomy

You might also like